2012 AMC 10/12 B Discussion
Go back to the Math Jam ArchiveA discussion of problems from the AMC 10/12 B, which is administered February 22. We will cover the last 5 problems on each test, as well as requested earlier problems on the tests.
Copyright © 2024 AoPS Incorporated. This page is copyrighted material. You can view and print this page for your own use, but you cannot share the contents of this file with others.
Facilitator: Jeremy Copeland
copeland
2012-02-23 19:03:06
Welcome to the 2012 AMC 10B/12B Math Jam!
Welcome to the 2012 AMC 10B/12B Math Jam!
copeland
2012-02-23 19:03:12
I'm Jeremy Copeland, and I'll be leading our discussion tonight.
I'm Jeremy Copeland, and I'll be leading our discussion tonight.
copeland
2012-02-23 19:03:17
Before we get started I would like to take a moment to explain our virtual classroom procedures to those who have not previously participated in a Math Jam or one of our online classes.
Before we get started I would like to take a moment to explain our virtual classroom procedures to those who have not previously participated in a Math Jam or one of our online classes.
copeland
2012-02-23 19:03:23
The classroom is moderated, meaning that students can type into the classroom, but these comments will not go directly into the room. These comments go to the instructors, who may choose to share your comments with the room.
The classroom is moderated, meaning that students can type into the classroom, but these comments will not go directly into the room. These comments go to the instructors, who may choose to share your comments with the room.
copeland
2012-02-23 19:03:31
This helps keep the class organized and on track. This also means that only well-written comments will be dropped into the classroom, so please take time writing responses that are complete and easy to read.
This helps keep the class organized and on track. This also means that only well-written comments will be dropped into the classroom, so please take time writing responses that are complete and easy to read.
copeland
2012-02-23 19:03:42
There are a lot of students here! As I said, only (a fraction of the) well-written comments will be passed to the entire group. Please do not take it personally if your comments do not get posted, and please do not complain about it. I expect this Math Jam to be much larger than our typical class, so please be patient with me---there are quite a few of you here tonight!!
There are a lot of students here! As I said, only (a fraction of the) well-written comments will be passed to the entire group. Please do not take it personally if your comments do not get posted, and please do not complain about it. I expect this Math Jam to be much larger than our typical class, so please be patient with me---there are quite a few of you here tonight!!
AkshajK
2012-02-23 19:03:51
you type fast
you type fast
copeland
2012-02-23 19:03:53
I have two keyboards.
I have two keyboards.
copeland
2012-02-23 19:03:58
Also, we won't be going through the math quite as thoroughly as we do in our classes -- I can't teach all the prerequisite material for every problem as we go. Another difference between tonight and our regular online classes is that it is very unlikely that we'll be able to answer every single question you ask. We usually do in our classes, but we have a large number of students tonight! So, please go ahead and ask questions, but also please understand if we aren't able to answer them all!
Also, we won't be going through the math quite as thoroughly as we do in our classes -- I can't teach all the prerequisite material for every problem as we go. Another difference between tonight and our regular online classes is that it is very unlikely that we'll be able to answer every single question you ask. We usually do in our classes, but we have a large number of students tonight! So, please go ahead and ask questions, but also please understand if we aren't able to answer them all!
copeland
2012-02-23 19:04:29
We do have two assistants tonight who can help answer some of your questions: Samson Zhou (TowersFreak2006) and Joel Schneider (joelinia) .
We do have two assistants tonight who can help answer some of your questions: Samson Zhou (TowersFreak2006) and Joel Schneider (joelinia) .
joelinia
2012-02-23 19:04:39
Hello! :-D
Hello! :-D
towersfreak2006
2012-02-23 19:04:40
Hi!
Hi!
basketballstar24
2012-02-23 19:04:52
Hi!
Hi!
rlingineni
2012-02-23 19:04:52
Hi guys :)
Hi guys :)
duketip10
2012-02-23 19:04:52
HI!
HI!
tb1624
2012-02-23 19:04:52
HIII!
HIII!
SHARKYBOY
2012-02-23 19:04:52
hi
hi
copeland
2012-02-23 19:04:58
They can answer questions by whispering to you or by opening a window with you to chat 1-on-1. However, due to the large size of the session tonight, they may not be able to get to you right away (or at all). Repeating your question over and over is more likely to annoy us than to get it answered faster, so please, just ask your question once and be patient, and please understand that we may not be able to answer all the questions tonight.
They can answer questions by whispering to you or by opening a window with you to chat 1-on-1. However, due to the large size of the session tonight, they may not be able to get to you right away (or at all). Repeating your question over and over is more likely to annoy us than to get it answered faster, so please, just ask your question once and be patient, and please understand that we may not be able to answer all the questions tonight.
copeland
2012-02-23 19:05:08
Please also remember that the purpose of this Math Jam is to work through the solutions to AMC problems, and not to merely present the answers. "Working through the solutions" includes discussing problem-solving tactics. So please, when a question is posted, do not simply respond with the final answer. That's not why we're here. We're going to work through the problems step-by-step, and comments that skip key steps or jump ahead in the problem, without providing explanation or motivation, won't be posted.
Please also remember that the purpose of this Math Jam is to work through the solutions to AMC problems, and not to merely present the answers. "Working through the solutions" includes discussing problem-solving tactics. So please, when a question is posted, do not simply respond with the final answer. That's not why we're here. We're going to work through the problems step-by-step, and comments that skip key steps or jump ahead in the problem, without providing explanation or motivation, won't be posted.
copeland
2012-02-23 19:05:43
We will work the last 5 problems from the AMC 10B, then 4 of the last 5 problems from the AMC 12B. (There's an overlap: #25 on the 10B is also #22 on the 12B.) After that, time permitting, I will take requests for some other problems for discussion.
We will work the last 5 problems from the AMC 10B, then 4 of the last 5 problems from the AMC 12B. (There's an overlap: #25 on the 10B is also #22 on the 12B.) After that, time permitting, I will take requests for some other problems for discussion.
copeland
2012-02-23 19:05:51
Let's get started!
Let's get started!
teethpaste
2012-02-23 19:06:10
yay!
yay!
MathForFun
2012-02-23 19:06:10
YAY!!
YAY!!
rlingineni
2012-02-23 19:06:10
YAY!!!
YAY!!!
copeland
2012-02-23 19:06:22
You'll notice that I'll always "sticky" the current problem under discussion to the top of the classroom window. You can resize the top region by dragging the horizontal gray bar that separates the top region from the main part of the classroom.
You'll notice that I'll always "sticky" the current problem under discussion to the top of the classroom window. You can resize the top region by dragging the horizontal gray bar that separates the top region from the main part of the classroom.
copeland
2012-02-23 19:06:31
Which number stands out to you and why?
Which number stands out to you and why?
Garchomp293
2012-02-23 19:07:10
2a, because it is different from the other a's.
2a, because it is different from the other a's.
christerson
2012-02-23 19:07:10
a + a = 2a
a + a = 2a
sammyMaX
2012-02-23 19:07:10
2a, because it is twice of many of the other sidelengths
2a, because it is twice of many of the other sidelengths
martian179
2012-02-23 19:07:10
2a because three points must be collinear to work
2a because three points must be collinear to work
copeland
2012-02-23 19:07:21
(I'll label the points X, Y, Z, and W.)
(I'll label the points X, Y, Z, and W.)
copeland
2012-02-23 19:07:40
Why is this interesting?
Why is this interesting?
alex31415
2012-02-23 19:08:16
but the triangle is degenerate
but the triangle is degenerate
AruKasera
2012-02-23 19:08:16
but it's degenarate!!
but it's degenarate!!
Seedleaf
2012-02-23 19:08:16
a + a = 2a, so they lie in a line.
a + a = 2a, so they lie in a line.
mcdonalds106_7
2012-02-23 19:08:16
three points must be collinear
three points must be collinear
swimmer
2012-02-23 19:08:16
the "triangle" is degenerate
the "triangle" is degenerate
briantix
2012-02-23 19:08:16
they form a line
they form a line
RotomPlasma
2012-02-23 19:08:16
Because then three of the points are collinear
Because then three of the points are collinear
negativebplusorminus
2012-02-23 19:08:16
A triangle like that is degenerate.
A triangle like that is degenerate.
mathman500
2012-02-23 19:08:16
degenerate triangle
degenerate triangle
TeamJacob
2012-02-23 19:08:16
because that's just a line segment, degenerate triangle
because that's just a line segment, degenerate triangle
copeland
2012-02-23 19:08:21
That triangle is degenerate. Three of the points are collinear.
That triangle is degenerate. Three of the points are collinear.
copeland
2012-02-23 19:08:27
Now how do we find W?
Now how do we find W?
somanywindows
2012-02-23 19:09:11
equilateral triangle
equilateral triangle
mcdonalds106_7
2012-02-23 19:09:11
it must be a away from 2 points
it must be a away from 2 points
Macromiidae
2012-02-23 19:09:11
equilateral triangle
equilateral triangle
willwang123
2012-02-23 19:09:11
there's an equilateral triangle
there's an equilateral triangle
martian179
2012-02-23 19:09:11
must be equidistant to two of other points
must be equidistant to two of other points
Flamel
2012-02-23 19:09:11
forms equilateral 'a' triangle
forms equilateral 'a' triangle
Iggy Iguana
2012-02-23 19:09:11
equalateral triangle
equalateral triangle
Jasonion
2012-02-23 19:09:11
make it equidistant from two of the points
make it equidistant from two of the points
HydraPyros
2012-02-23 19:09:11
Choose two adjacent points, that will be the base of an equilateral triangle of side length a. W is the vertex of this triangle.
Choose two adjacent points, that will be the base of an equilateral triangle of side length a. W is the vertex of this triangle.
swimmer
2012-02-23 19:09:11
it forms an equilateral triangle with two of the other points
it forms an equilateral triangle with two of the other points
copeland
2012-02-23 19:09:14
Since we still have 2 segments from W that are of length a, W is part of an equilateral triangle. By symmetry we can assume X, Y, and W are the vertices of the equilateral triangle.
Since we still have 2 segments from W that are of length a, W is part of an equilateral triangle. By symmetry we can assume X, Y, and W are the vertices of the equilateral triangle.
copeland
2012-02-23 19:09:22
What is b/a?
What is b/a?
mcqueen
2012-02-23 19:10:12
/whisper BigSams I can't see the problems. They're just blank message text. Any help?
/whisper BigSams I can't see the problems. They're just blank message text. Any help?
Seedleaf
2012-02-23 19:10:12
sqrt3
sqrt3
teethpaste
2012-02-23 19:10:12
sqrt3
sqrt3
Lalagato
2012-02-23 19:10:12
rt(3)
rt(3)
christerson
2012-02-23 19:10:12
sqrt{3}
sqrt{3}
tan90
2012-02-23 19:10:12
sqrt3
sqrt3
sumkid25
2012-02-23 19:10:12
sqrt(3)
sqrt(3)
sammyMaX
2012-02-23 19:10:12
sqrt3
sqrt3
HydraPyros
2012-02-23 19:10:12
sqrt(3)
sqrt(3)
NeilOnnsu
2012-02-23 19:10:12
sqrt(3)
sqrt(3)
tc1729
2012-02-23 19:10:12
sqrt(3)
sqrt(3)
mathocean97
2012-02-23 19:10:12
\sqrt(3)
\sqrt(3)
AruKasera
2012-02-23 19:10:12
sqrt3!
sqrt3!
.cpp
2012-02-23 19:10:12
sqrt(3)
sqrt(3)
azure55
2012-02-23 19:10:12
rt3
rt3
negativebplusorminus
2012-02-23 19:10:13
By the Law of Cosines, b^2=3a^2, so the answer is Sqrt[3]; A.
By the Law of Cosines, b^2=3a^2, so the answer is Sqrt[3]; A.
hali323
2012-02-23 19:10:13
sqrt3
sqrt3
Xu12345
2012-02-23 19:10:13
sqrt 3
sqrt 3
lucylai
2012-02-23 19:10:13
sqrt3
sqrt3
duketip10
2012-02-23 19:10:14
sqrt3
sqrt3
copeland
2012-02-23 19:10:32
The answer is (A).
The answer is (A).
copeland
2012-02-23 19:10:57
(I likely won't pass that many answers in the future. There are a LOT of people here, though, so don't get sad if what you say doesn't get passed!)
(I likely won't pass that many answers in the future. There are a LOT of people here, though, so don't get sad if what you say doesn't get passed!)
copeland
2012-02-23 19:11:51
More!
More!
copeland
2012-02-23 19:12:01
If you haven't seen this problem before, it's a little hard to know where to start.
If you haven't seen this problem before, it's a little hard to know where to start.
copeland
2012-02-23 19:12:04
What should we try here?
What should we try here?
.cpp
2012-02-23 19:12:38
Small cases.
Small cases.
christerson
2012-02-23 19:12:38
Small cases
Small cases
martian179
2012-02-23 19:12:38
start with first number is 1
start with first number is 1
negativebplusorminus
2012-02-23 19:12:38
Smaller cases?
Smaller cases?
AkshajK
2012-02-23 19:12:38
making the problem easier
making the problem easier
VIPMaster
2012-02-23 19:12:38
Smaller cases, in an attempt to find a recursion
Smaller cases, in an attempt to find a recursion
ABCDE
2012-02-23 19:12:38
Smaller cases.
Smaller cases.
guilt
2012-02-23 19:12:38
Small cases.
Small cases.
copeland
2012-02-23 19:12:43
10 is a really big number. Let's start with something smaller. How many sequences like this are there using only the digits 1 and 2?
10 is a really big number. Let's start with something smaller. How many sequences like this are there using only the digits 1 and 2?
guilt
2012-02-23 19:13:14
2
2
Jasmine8925
2012-02-23 19:13:14
12, 21
12, 21
swimmer
2012-02-23 19:13:14
2
2
VIPMaster
2012-02-23 19:13:14
2
2
wcao9311
2012-02-23 19:13:14
2
2
martian179
2012-02-23 19:13:14
2
2
mcdonalds106_7
2012-02-23 19:13:14
2.
2.
numbertheory
2012-02-23 19:13:14
2
2
hi how are you doing toda
2012-02-23 19:13:14
2
2
.cpp
2012-02-23 19:13:14
2: (1,2) or (2,1)
2: (1,2) or (2,1)
mcdonalds106_7
2012-02-23 19:13:14
1,2 and 2,1
1,2 and 2,1
richard4912
2012-02-23 19:13:14
2
2
copeland
2012-02-23 19:13:16
Both sequences 12 and 21 satisfy the hypothesis.
Both sequences 12 and 21 satisfy the hypothesis.
copeland
2012-02-23 19:13:20
What about with the digits 1, 2, and 3?
What about with the digits 1, 2, and 3?
guilt
2012-02-23 19:13:57
4
4
newchie123
2012-02-23 19:13:57
4
4
christerson
2012-02-23 19:13:57
4
4
Luckytoilet
2012-02-23 19:13:57
123, 321, 213, 231
123, 321, 213, 231
mcdonalds106_7
2012-02-23 19:13:57
4- 1,2,3 2,1,3 2,3,1 3,2,1
4- 1,2,3 2,1,3 2,3,1 3,2,1
teethpaste
2012-02-23 19:13:57
4
4
alex31415
2012-02-23 19:13:57
132, 312 are illegal=>4
132, 312 are illegal=>4
RotomPlasma
2012-02-23 19:13:57
4
4
.cpp
2012-02-23 19:13:57
4: (1,2,3), (2,1,3), (2,3,1), or (3,2,1)
4: (1,2,3), (2,1,3), (2,3,1), or (3,2,1)
lucylai
2012-02-23 19:13:57
4
4
Iggy Iguana
2012-02-23 19:13:57
(1, 2, 3) (2, 3, 1) (2, 1, 3) (3, 2, 1)
(1, 2, 3) (2, 3, 1) (2, 1, 3) (3, 2, 1)
AkshajK
2012-02-23 19:13:57
4
4
pjyoung
2012-02-23 19:13:57
123 132 213 231 312 321
123 132 213 231 312 321
willwang123
2012-02-23 19:13:59
123, 213, 231, 321
123, 213, 231, 321
copeland
2012-02-23 19:14:03
Working the other direction, how many sequences are there that use just the number 1?
Working the other direction, how many sequences are there that use just the number 1?
pr0likethis
2012-02-23 19:14:25
1
1
briantix
2012-02-23 19:14:25
1
1
teethpaste
2012-02-23 19:14:25
1
1
MCL
2012-02-23 19:14:25
1.
1.
joshxiong
2012-02-23 19:14:25
1
1
Jasonion
2012-02-23 19:14:25
1
1
alex31415
2012-02-23 19:14:25
1
1
sammyMaX
2012-02-23 19:14:25
1!
1!
numbertheory
2012-02-23 19:14:25
1
1
coldsummer
2012-02-23 19:14:25
1
1
lucylai
2012-02-23 19:14:25
1
1
somanywindows
2012-02-23 19:14:25
1
1
copeland
2012-02-23 19:14:30
There is one such sequence, the sequence 1.
There is one such sequence, the sequence 1.
copeland
2012-02-23 19:14:33
What about with the digits 1, 2, 3, and 4?
What about with the digits 1, 2, 3, and 4?
ABCDE
2012-02-23 19:14:56
8
8
christerson
2012-02-23 19:14:56
8
8
va2010
2012-02-23 19:14:56
there are 8
there are 8
super_pi314
2012-02-23 19:14:56
8
8
NeilOnnsu
2012-02-23 19:14:56
8
8
briantix
2012-02-23 19:14:56
8
8
hali323
2012-02-23 19:14:56
there are 8
there are 8
MCL
2012-02-23 19:14:56
8.
8.
joshxiong
2012-02-23 19:14:56
8
8
copeland
2012-02-23 19:15:05
There are 8 of these.
There are 8 of these.
copeland
2012-02-23 19:15:08
Now we can probably make a reasonable guess. Our sequence starts 1, 2, 4, 8,. . . .
Now we can probably make a reasonable guess. Our sequence starts 1, 2, 4, 8,. . . .
copeland
2012-02-23 19:15:15
What ought to be the 10th term in this sequence?
What ought to be the 10th term in this sequence?
MathForFun
2012-02-23 19:15:44
16, 32, 64... 512
16, 32, 64... 512
willwang123
2012-02-23 19:15:44
2^9 = 512
2^9 = 512
Iggy Iguana
2012-02-23 19:15:44
2^9=512
2^9=512
AkshajK
2012-02-23 19:15:44
(B) 512
(B) 512
Seedleaf
2012-02-23 19:15:44
2^9 = 512
2^9 = 512
martian179
2012-02-23 19:15:44
2^9=512
2^9=512
coldsummer
2012-02-23 19:15:44
512
512
Mattchu386
2012-02-23 19:15:44
512
512
sammyMaX
2012-02-23 19:15:44
512
512
Klu2014
2012-02-23 19:15:44
2^9
2^9
numbertheory
2012-02-23 19:15:44
512
512
MCL
2012-02-23 19:16:06
How do we prove this?
How do we prove this?
mentalgenius
2012-02-23 19:16:06
I see a pattern (powers of 2), but how do we mathematically make sure that this is true?
I see a pattern (powers of 2), but how do we mathematically make sure that this is true?
swimmer
2012-02-23 19:16:06
how do you prove this though?
how do you prove this though?
copeland
2012-02-23 19:16:15
Good! Thinking like mathematicians now.
Good! Thinking like mathematicians now.
copeland
2012-02-23 19:16:30
How should we approach this?
How should we approach this?
alex31415
2012-02-23 19:17:08
prove the recursive pattern
prove the recursive pattern
Iggy Iguana
2012-02-23 19:17:08
recursion, induction
recursion, induction
negativebplusorminus
2012-02-23 19:17:08
Recursion!
Recursion!
superpi83
2012-02-23 19:17:08
induction?
induction?
joshxiong
2012-02-23 19:17:08
Induction, maybe
Induction, maybe
newchie123
2012-02-23 19:17:08
INDUCTION
INDUCTION
iwantcombo
2012-02-23 19:17:08
induction?
induction?
copeland
2012-02-23 19:17:13
Let's work recursively.
Let's work recursively.
copeland
2012-02-23 19:17:14
Why are there always twice as many terms in an n-term sequence as there are in an (n-1)-term sequence?
Why are there always twice as many terms in an n-term sequence as there are in an (n-1)-term sequence?
copeland
2012-02-23 19:17:39
Let's look at our 8 again:
Let's look at our 8 again:
copeland
2012-02-23 19:17:48
What do you notice about all these sequences?
What do you notice about all these sequences?
.cpp
2012-02-23 19:18:23
End with 4 or 1.
End with 4 or 1.
super_pi314
2012-02-23 19:18:23
they end in 1 or 4
they end in 1 or 4
Iggy Iguana
2012-02-23 19:18:23
1 or 4 is at the end
1 or 4 is at the end
MCL
2012-02-23 19:18:23
they end in 4 or 1
they end in 4 or 1
apple.singer
2012-02-23 19:18:23
all end in 1/4
all end in 1/4
myseokim
2012-02-23 19:18:23
ends in 1 or 4
ends in 1 or 4
fortenforge
2012-02-23 19:18:23
the last digit is either a 4 or a 1
the last digit is either a 4 or a 1
VIPMaster
2012-02-23 19:18:23
they end with either 1 or 4
they end with either 1 or 4
copeland
2012-02-23 19:18:27
We can start with any number we want. Then each new number must be either (the lowest number that's greater than all the numbers we've placed) or (the highest number that's less than all the numbers we've placed). (E.g. at some intermediate step we might have placed all of 3 through 8. The next number must be 2 or 9.) At the end this will leave us with either 1 or n.
We can start with any number we want. Then each new number must be either (the lowest number that's greater than all the numbers we've placed) or (the highest number that's less than all the numbers we've placed). (E.g. at some intermediate step we might have placed all of 3 through 8. The next number must be 2 or 9.) At the end this will leave us with either 1 or n.
copeland
2012-02-23 19:18:40
So. . . .
So. . . .
lucylai
2012-02-23 19:20:18
you can just attach a 4 to the end of a rearrangement of n-1
you can just attach a 4 to the end of a rearrangement of n-1
negativebplusorminus
2012-02-23 19:20:18
So if we delete that number, we either have the sequences for 9, or each one for 9, plus one.
So if we delete that number, we either have the sequences for 9, or each one for 9, plus one.
copeland
2012-02-23 19:20:40
In particular we can build a sequence from right-to-left by placing either 1 or n first, and then in every step picking either the highest or lowest term that we haven't placed yet. There are 2^{n-1} ways to do this because there is no choice for the final digit placed.
In particular we can build a sequence from right-to-left by placing either 1 or n first, and then in every step picking either the highest or lowest term that we haven't placed yet. There are 2^{n-1} ways to do this because there is no choice for the final digit placed.
copeland
2012-02-23 19:21:10
So we just proved that the answer is. . . .
So we just proved that the answer is. . . .
Klu2014
2012-02-23 19:21:35
512
512
iwantcombo
2012-02-23 19:21:35
B
B
swimmer
2012-02-23 19:21:35
512
512
teethpaste
2012-02-23 19:21:35
512
512
ap1998
2012-02-23 19:21:35
512
512
mathgenius1982
2012-02-23 19:21:35
2^9=512
2^9=512
mcdonalds106_7
2012-02-23 19:21:35
512
512
christerson
2012-02-23 19:21:35
512
512
copeland
2012-02-23 19:21:38
(B) 512.
(B) 512.
copeland
2012-02-23 19:21:54
Good job. Let's see more!
Good job. Let's see more!
copeland
2012-02-23 19:22:21
Rule number 1 for a problem like this is. . . .
Rule number 1 for a problem like this is. . . .
googol.plex
2012-02-23 19:22:51
Draw it!
Draw it!
alex31415
2012-02-23 19:22:51
Draw a diagram
Draw a diagram
mentalgenius
2012-02-23 19:22:51
Diagram please?
Diagram please?
pr0likethis
2012-02-23 19:22:51
draw it out
draw it out
superpi83
2012-02-23 19:22:51
draw it?
draw it?
Turtle
2012-02-23 19:22:51
drawing
drawing
AmericanPi
2012-02-23 19:22:51
draw a diagram
draw a diagram
va2010
2012-02-23 19:22:51
Draw a picture
Draw a picture
sparkles257
2012-02-23 19:22:51
draw a diagram
draw a diagram
Cortana
2012-02-23 19:22:51
Picture
Picture
Draco
2012-02-23 19:22:51
Draw a picture!
Draw a picture!
etothei
2012-02-23 19:22:51
draw a picture!
draw a picture!
ken961996
2012-02-23 19:22:51
diagram
diagram
copeland
2012-02-23 19:22:55
Draw a diagram!
Draw a diagram!
copeland
2012-02-23 19:22:59
Here's a cube:
Here's a cube:
copeland
2012-02-23 19:23:06
Here are three vertices that fit the bill:
Here are three vertices that fit the bill:
copeland
2012-02-23 19:23:18
We're supposed to cut an rotate to get this picture:
We're supposed to cut an rotate to get this picture:
copeland
2012-02-23 19:23:35
We want to know the height, so we want to know the distance from the top vertex to the red plane. How can we compute this height?
We want to know the height, so we want to know the distance from the top vertex to the red plane. How can we compute this height?
iwantcombo
2012-02-23 19:24:18
coordinate bash
coordinate bash
mathdate
2012-02-23 19:24:18
the vertex to the centroid of the triangle
the vertex to the centroid of the triangle
sparkles257
2012-02-23 19:24:18
find heigth of the tetrahedron and then subtract from psace daigonal
find heigth of the tetrahedron and then subtract from psace daigonal
lucylai
2012-02-23 19:24:18
subtract height of tetrahedron from diagonal length
subtract height of tetrahedron from diagonal length
mentalgenius
2012-02-23 19:24:18
subtract the altitude of the tetrahedron from the space diag of the cube (sqrt(3))
subtract the altitude of the tetrahedron from the space diag of the cube (sqrt(3))
mcdonalds106_7
2012-02-23 19:24:18
space diagonal of cube minus height of tetrahedron that was cut off
space diagonal of cube minus height of tetrahedron that was cut off
NeilOnnsu
2012-02-23 19:24:18
find interior diagnol - height of tetrahedron
find interior diagnol - height of tetrahedron
iwantcombo
2012-02-23 19:24:18
or drop a perpendicular that hits the centroid of the triangle
or drop a perpendicular that hits the centroid of the triangle
va2010
2012-02-23 19:24:18
Take the diagnal and substract the pyramids height
Take the diagnal and substract the pyramids height
c0mbusti0n1295
2012-02-23 19:24:18
height of the cube?
height of the cube?
TeamJacob
2012-02-23 19:24:18
drawing a triangular pyramid using that base
drawing a triangular pyramid using that base
iwantcombo
2012-02-23 19:24:18
then take the space diagonal and subtract
then take the space diagonal and subtract
copeland
2012-02-23 19:24:23
That's a few ideas.
That's a few ideas.
copeland
2012-02-23 19:24:28
We had a few ideas here, too.
We had a few ideas here, too.
copeland
2012-02-23 19:24:35
The most "geometric" approach is by computing the altitude of the tetrahedron we cut off and subtracting.
The most "geometric" approach is by computing the altitude of the tetrahedron we cut off and subtracting.
copeland
2012-02-23 19:24:44
How can we get at that altitude?
How can we get at that altitude?
tan90
2012-02-23 19:25:18
Using volume of pyramid
Using volume of pyramid
Relativity1618
2012-02-23 19:25:18
finding volume
finding volume
.cpp
2012-02-23 19:25:18
Find volume two ways.
Find volume two ways.
Relativity1618
2012-02-23 19:25:18
finding volume
finding volume
munygrubber
2012-02-23 19:25:18
find the volume of the pyramid in two ways
find the volume of the pyramid in two ways
fortenforge
2012-02-23 19:25:18
Find the volume of the tetrahedron and divide by the base area
Find the volume of the tetrahedron and divide by the base area
RotomPlasma
2012-02-23 19:25:18
It's a pyramid, with known base, and we know the volume
It's a pyramid, with known base, and we know the volume
copeland
2012-02-23 19:25:22
We can compute the volume of the tetrahedron in 2 ways. Let's go back to this picture:
We can compute the volume of the tetrahedron in 2 ways. Let's go back to this picture:
copeland
2012-02-23 19:25:29
Numerically what is the volume of the tetrahedron?
Numerically what is the volume of the tetrahedron?
christerson
2012-02-23 19:26:01
1/6
1/6
mcdonalds106_7
2012-02-23 19:26:01
1/6
1/6
duketip10
2012-02-23 19:26:01
1/6
1/6
ss5188
2012-02-23 19:26:01
1/6
1/6
.cpp
2012-02-23 19:26:01
1/3 * 1/2 = 1/6
1/3 * 1/2 = 1/6
msinghal
2012-02-23 19:26:01
1/2*1*1/3=1/6
1/2*1*1/3=1/6
PChang
2012-02-23 19:26:01
1/6
1/6
copeland
2012-02-23 19:26:05
The tetrahedron has triangular base that is half a face of the cube and has height equal to 1. Therefore it has volume 1/6.
The tetrahedron has triangular base that is half a face of the cube and has height equal to 1. Therefore it has volume 1/6.
copeland
2012-02-23 19:26:09
In terms of the altitude we care about, what is the volume?
In terms of the altitude we care about, what is the volume?
alex31415
2012-02-23 19:27:03
1/3*b*h
1/3*b*h
Relativity1618
2012-02-23 19:27:03
H*B/3
H*B/3
Iggy Iguana
2012-02-23 19:27:03
sqrt3/2 * h / 3
sqrt3/2 * h / 3
tan90
2012-02-23 19:27:03
Bh/3
Bh/3
Relativity1618
2012-02-23 19:27:03
volume=height times base divided by 3
volume=height times base divided by 3
.cpp
2012-02-23 19:27:03
1/3 * sqrt{3}/2 * h
1/3 * sqrt{3}/2 * h
swimmer
2012-02-23 19:27:03
h * sqrt(3)/6
h * sqrt(3)/6
lucylai
2012-02-23 19:27:03
sqrt3/6*altitude
sqrt3/6*altitude
coldsummer
2012-02-23 19:27:03
1/2 basex height
1/2 basex height
copeland
2012-02-23 19:27:15
What is the value of h?
What is the value of h?
briantix
2012-02-23 19:28:04
sqrt3 /3
sqrt3 /3
RotomPlasma
2012-02-23 19:28:04
(sqrt 3)/3
(sqrt 3)/3
christerson
2012-02-23 19:28:04
sqrt(3)/3
sqrt(3)/3
PChang
2012-02-23 19:28:04
sqrt(3)/3
sqrt(3)/3
ss5188
2012-02-23 19:28:04
rad3 /3
rad3 /3
NeilOnnsu
2012-02-23 19:28:04
sqrt(3)/3
sqrt(3)/3
googol.plex
2012-02-23 19:28:04
\sqrt {3}/3
\sqrt {3}/3
va2010
2012-02-23 19:28:04
sqrt(3)/3
sqrt(3)/3
Garchomp293
2012-02-23 19:28:04
1/sqrt3
1/sqrt3
ddot1
2012-02-23 19:28:04
sqrt{3}/3
sqrt{3}/3
mentalgenius
2012-02-23 19:28:04
1/sqrt(3)
1/sqrt(3)
sparkles257
2012-02-23 19:28:04
sqrt3/3
sqrt3/3
copeland
2012-02-23 19:28:16
So what's the answer?
So what's the answer?
tan90
2012-02-23 19:28:48
(D)
(D)
Relativity1618
2012-02-23 19:28:48
D
D
msinghal
2012-02-23 19:28:48
D
D
negativebplusorminus
2012-02-23 19:28:48
So, since the space diagonal is Sqrt[3], the answer is D.
So, since the space diagonal is Sqrt[3], the answer is D.
sparkles257
2012-02-23 19:28:48
subtract that from sqrt 3 to get 2sqrt3/3 D
subtract that from sqrt 3 to get 2sqrt3/3 D
Glaydus
2012-02-23 19:28:48
D
D
Iggy Iguana
2012-02-23 19:28:48
sqrt3 - sqrt3/3 = 2sqrt3/3
sqrt3 - sqrt3/3 = 2sqrt3/3
MNL9082
2012-02-23 19:28:48
2sqrt(3)/3
2sqrt(3)/3
joshxiong
2012-02-23 19:28:48
D. 2sqrt3/3
D. 2sqrt3/3
Relativity1618
2012-02-23 19:28:48
D
D
iwantcombo
2012-02-23 19:28:48
D
D
copeland
2012-02-23 19:28:50
You guys just cannot be tricked!
You guys just cannot be tricked!
copeland
2012-02-23 19:29:00
h is not what we want. We want the height that's left once we cut that tetrahedron off.
h is not what we want. We want the height that's left once we cut that tetrahedron off.
copeland
2012-02-23 19:29:08
The answer is (D).
The answer is (D).
mcdonalds106_7
2012-02-23 19:29:22
what were your other ideas?
what were your other ideas?
copeland
2012-02-23 19:29:24
This is not how I solved the problem.
This is not how I solved the problem.
copeland
2012-02-23 19:29:36
I like coordinates with cubec.
I like coordinates with cubec.
copeland
2012-02-23 19:29:38
cubes.
cubes.
copeland
2012-02-23 19:29:41
Cubes like coordinates.
Cubes like coordinates.
copeland
2012-02-23 19:29:44
A more efficient solution goes like this. We let the unit cube be all triples (a,b,c) where each of a, b, and c are either 0 or 1. Then we place (0,0,0) at the top of this figure:
A more efficient solution goes like this. We let the unit cube be all triples (a,b,c) where each of a, b, and c are either 0 or 1. Then we place (0,0,0) at the top of this figure:
copeland
2012-02-23 19:29:51
What are the three base vertices?
What are the three base vertices?
.cpp
2012-02-23 19:30:38
(1,1,0), (1,0,1), (0,1,1)
(1,1,0), (1,0,1), (0,1,1)
Bictor717
2012-02-23 19:30:38
two ones, one zero
two ones, one zero
nsun48
2012-02-23 19:30:38
110,101,001
110,101,001
superpi83
2012-02-23 19:30:38
(1,1,0) (1,0,1) (0,1,1)
(1,1,0) (1,0,1) (0,1,1)
tan90
2012-02-23 19:30:40
(1,1,0) (1,0,1) (0,1,1)
(1,1,0) (1,0,1) (0,1,1)
super_pi314
2012-02-23 19:30:40
(1, 0, 0); (0, 1, 0); (1,1,1)
(1, 0, 0); (0, 1, 0); (1,1,1)
swimmer
2012-02-23 19:30:40
(1,0,0), (0,1,0), (0,0,1)
(1,0,0), (0,1,0), (0,0,1)
Yongyi781
2012-02-23 19:30:40
(0, 1, 1) and permutations
(0, 1, 1) and permutations
azure55
2012-02-23 19:30:40
(1,0,0),(0,0,1),(0,1,0)
(1,0,0),(0,0,1),(0,1,0)
copeland
2012-02-23 19:30:43
The vertices of the base triangle are (1,1,0), (0,1,1), and (1,0,1).
The vertices of the base triangle are (1,1,0), (0,1,1), and (1,0,1).
copeland
2012-02-23 19:30:48
Where does the altitude meet the base triangle?
Where does the altitude meet the base triangle?
swimmer
2012-02-23 19:31:24
center
center
mentalgenius
2012-02-23 19:31:24
centroid
centroid
duketip10
2012-02-23 19:31:24
centroid
centroid
c0mbusti0n1295
2012-02-23 19:31:24
centroid?
centroid?
superpi83
2012-02-23 19:31:24
at the centroid of the base triangle, which is clearly (2/3,2/3,2/3)
at the centroid of the base triangle, which is clearly (2/3,2/3,2/3)
msinghal
2012-02-23 19:31:24
(2/3, 2/3, 2/3)
(2/3, 2/3, 2/3)
joshxiong
2012-02-23 19:31:24
At the centroid
At the centroid
AkshajK
2012-02-23 19:31:24
at 2/3, 2/3, 2/3
at 2/3, 2/3, 2/3
.cpp
2012-02-23 19:31:24
The very center (2/3, 2/3, 2/3).
The very center (2/3, 2/3, 2/3).
copeland
2012-02-23 19:31:32
What's the altitude then?
What's the altitude then?
nsun48
2012-02-23 19:32:11
we use 3d pythag to get the answer
we use 3d pythag to get the answer
googol.plex
2012-02-23 19:32:11
2/3*sqrt 3
2/3*sqrt 3
superpi83
2012-02-23 19:32:11
2sqrt(3)/3
2sqrt(3)/3
duketip10
2012-02-23 19:32:11
the magnitude
the magnitude
swimmer
2012-02-23 19:32:11
2sqrt3/3
2sqrt3/3
cerberus88
2012-02-23 19:32:11
2\sqrt{3}/3
2\sqrt{3}/3
somanywindows
2012-02-23 19:32:11
2sqrt3/3
2sqrt3/3
Garchomp293
2012-02-23 19:32:11
2sqrt3/3
2sqrt3/3
msinghal
2012-02-23 19:32:11
2/3*sqrt3
2/3*sqrt3
NeilOnnsu
2012-02-23 19:32:11
2/3*sqrt(3)
2/3*sqrt(3)
Yongyi781
2012-02-23 19:32:11
2sqrt(3)/3
2sqrt(3)/3
lucylai
2012-02-23 19:32:11
sqrt(8/9)=2sqrt3/3
sqrt(8/9)=2sqrt3/3
ddot1
2012-02-23 19:32:11
2sqrt{3}/3
2sqrt{3}/3
copeland
2012-02-23 19:32:18
That's what I did.
That's what I did.
copeland
2012-02-23 19:32:24
We had someone else in the office go this way:
We had someone else in the office go this way:
copeland
2012-02-23 19:32:28
Another approach is by using these same coordinates and noticing that the cut is on the plane x+y+z-2=0. Then apply the formula for the distance from a point to a plane and you're done.
Another approach is by using these same coordinates and noticing that the cut is on the plane x+y+z-2=0. Then apply the formula for the distance from a point to a plane and you're done.
copeland
2012-02-23 19:32:47
There's a prettier answer, though.
There's a prettier answer, though.
copeland
2012-02-23 19:33:03
I mean, why, after all, is this a rational multiple of the length of the diagonal?
I mean, why, after all, is this a rational multiple of the length of the diagonal?
copeland
2012-02-23 19:33:06
A fourth approach draws all the parallel planes x+y+z=n for integers n. These planes intersect the cube at the top vertex, the bottom vertex, and pass through trios of points. In particular one of these is the plane we are interested in.
A fourth approach draws all the parallel planes x+y+z=n for integers n. These planes intersect the cube at the top vertex, the bottom vertex, and pass through trios of points. In particular one of these is the plane we are interested in.
c0mbusti0n1295
2012-02-23 19:33:52
wait, sqrt 3/2?
wait, sqrt 3/2?
AkshajK
2012-02-23 19:33:52
you mean 2sqrt3/3
you mean 2sqrt3/3
negativebplusorminus
2012-02-23 19:33:52
Not Sqrt[3]/2.
Not Sqrt[3]/2.
copeland
2012-02-23 19:33:57
Yeah, typing is hard.
Yeah, typing is hard.
Yongyi781
2012-02-23 19:34:51
Why is it obvious that they trisect the space diagonal?
Why is it obvious that they trisect the space diagonal?
copeland
2012-02-23 19:34:53
There's another way to see this picture: line up a bunch of cubes in a row and cut them with parallel planes. That gives an explicit picture of this trisection by translation symmetry.
There's another way to see this picture: line up a bunch of cubes in a row and cut them with parallel planes. That gives an explicit picture of this trisection by translation symmetry.
copeland
2012-02-23 19:35:07
The fact that it's a trisection, though, follows from the fact that the planes are evenly spaced.
The fact that it's a trisection, though, follows from the fact that the planes are evenly spaced.
copeland
2012-02-23 19:35:27
Next!
Next!
copeland
2012-02-23 19:35:40
When I see a problem like this, I like to try to sketch an example. We have two sets of objects and we have relationships between them.
When I see a problem like this, I like to try to sketch an example. We have two sets of objects and we have relationships between them.
copeland
2012-02-23 19:35:54
Now we don't know which song is which and we don't know how many of the girls are fans of Happy Birthday (it could be 0, 1, or 2).
Now we don't know which song is which and we don't know how many of the girls are fans of Happy Birthday (it could be 0, 1, or 2).
copeland
2012-02-23 19:36:03
How do we attack this?
How do we attack this?
joshxiong
2012-02-23 19:36:36
Casework.
Casework.
alex31415
2012-02-23 19:36:36
casework
casework
MNL9082
2012-02-23 19:36:36
Use casework
Use casework
briantix
2012-02-23 19:36:36
casework!
casework!
Asmodeus1123
2012-02-23 19:36:36
casework?
casework?
va2010
2012-02-23 19:36:36
Casework
Casework
fishinsea
2012-02-23 19:36:36
casework?
casework?
Klu2014
2012-02-23 19:36:36
casework
casework
pgmath
2012-02-23 19:36:36
Casework.
Casework.
MathForFun
2012-02-23 19:36:36
Casework...
Casework...
swimdad11
2012-02-23 19:36:36
we could do organized casework.
we could do organized casework.
copeland
2012-02-23 19:36:39
We should use casework on the number of girls who liked the fourth song.
We should use casework on the number of girls who liked the fourth song.
copeland
2012-02-23 19:36:43
I actually drew the less useful graph up there. What should we draw instead?
I actually drew the less useful graph up there. What should we draw instead?
superpi83
2012-02-23 19:37:41
graph of what they don't like
graph of what they don't like
.cpp
2012-02-23 19:37:41
Connect dislikes.
Connect dislikes.
lucylai
2012-02-23 19:37:41
like-dislike diagrams
like-dislike diagrams
copeland
2012-02-23 19:37:45
Let's pair the songs with the girls that dislike that song. Our example above becomes:
Let's pair the songs with the girls that dislike that song. Our example above becomes:
copeland
2012-02-23 19:38:03
Our cases are whether 1, 2, or 3 girls dislike the last song.
Our cases are whether 1, 2, or 3 girls dislike the last song.
copeland
2012-02-23 19:38:09
Let's start with the case that all 3 girls dislike one of the songs. How many ways are there to build a graph satisfying all the hypotheses where all three girls dislike one of the songs? (and not necessarily such that Happy Birthday is the despicable song.)
Let's start with the case that all 3 girls dislike one of the songs. How many ways are there to build a graph satisfying all the hypotheses where all three girls dislike one of the songs? (and not necessarily such that Happy Birthday is the despicable song.)
flyrain
2012-02-23 19:39:04
24
24
[cube]
2012-02-23 19:39:04
4!
4!
christerson
2012-02-23 19:39:04
24
24
.cpp
2012-02-23 19:39:04
4*4!
4*4!
zheng
2012-02-23 19:39:04
24 since the songs can be ordered in any way
24 since the songs can be ordered in any way
briantix
2012-02-23 19:39:04
24
24
joshxiong
2012-02-23 19:39:04
24.
24.
moose97
2012-02-23 19:39:04
4! ways
4! ways
AkshajK
2012-02-23 19:39:04
There are 24, 4 ways to pick what joe dislikes, 3 ways to think what bath dislikes, and 2 ways to blink what Emi dislikes.
There are 24, 4 ways to pick what joe dislikes, 3 ways to think what bath dislikes, and 2 ways to blink what Emi dislikes.
Seedleaf
2012-02-23 19:39:04
24 ways
24 ways
superpi83
2012-02-23 19:39:05
pick 1 song to be the despicable song (4 ways), then assign the remaining songs to the three girls (3!=6 ways); total 24 ways
pick 1 song to be the despicable song (4 ways), then assign the remaining songs to the three girls (3!=6 ways); total 24 ways
somanywindows
2012-02-23 19:39:05
24
24
copeland
2012-02-23 19:39:09
There are 4 ways to pick the universally reviled song. Then, since there still needs to be a unique song distasteful to each of the three girls, there are 3! ways to pair the three girls with the remaining three girls. This gives a total of 24 graphs.
There are 4 ways to pick the universally reviled song. Then, since there still needs to be a unique song distasteful to each of the three girls, there are 3! ways to pair the three girls with the remaining three girls. This gives a total of 24 graphs.
copeland
2012-02-23 19:39:13
How many graphs can we draw where 2 of the girls hate one of the songs?
How many graphs can we draw where 2 of the girls hate one of the songs?
briantix
2012-02-23 19:40:13
72
72
somanywindows
2012-02-23 19:40:13
72
72
Seedleaf
2012-02-23 19:40:13
72 ways
72 ways
Luckytoilet
2012-02-23 19:40:13
24*3
24*3
negativebplusorminus
2012-02-23 19:40:13
72
72
superpi83
2012-02-23 19:40:13
4 ways to pick the hated song, 3 ways to pick the 2 haters of the hated song, then 6 ways to do the rest. So 72.
4 ways to pick the hated song, 3 ways to pick the 2 haters of the hated song, then 6 ways to do the rest. So 72.
cerberus88
2012-02-23 19:40:13
48
48
tan90
2012-02-23 19:40:13
72
72
zheng
2012-02-23 19:40:13
3*24=72 since we must now orderthe girls also
3*24=72 since we must now orderthe girls also
copeland
2012-02-23 19:40:17
There are still 4 ways to pick doubly-hated song, but now there are 3 ways to pick the unique girl who likes that song. After that we still have 3! ways to assign the remaining songs. That gives 72 graphs with a doubly-hated song.
There are still 4 ways to pick doubly-hated song, but now there are 3 ways to pick the unique girl who likes that song. After that we still have 3! ways to assign the remaining songs. That gives 72 graphs with a doubly-hated song.
copeland
2012-02-23 19:40:25
How many graphs can we find where all of the songs are unpalatable to exactly one of the girls?
How many graphs can we find where all of the songs are unpalatable to exactly one of the girls?
copeland
2012-02-23 19:40:32
Now an example of such a graph is
Now an example of such a graph is
lucylai
2012-02-23 19:41:37
3*12=36
3*12=36
Seedleaf
2012-02-23 19:41:37
36 ways
36 ways
TeamJacob
2012-02-23 19:41:37
36
36
briantix
2012-02-23 19:41:37
36
36
Iggy Iguana
2012-02-23 19:41:37
3 * 4 * 3 = 36
3 * 4 * 3 = 36
AkshajK
2012-02-23 19:41:37
4 ways to pick what joe dislikes, 3 ways to think what bath dislikes, and 2 ways to blink what Emi dislikes. Now, 3 ways to pick the pair that likes the last song, BUT we have to divide by 2 because those same two also like another song, and we don't want to overcount. So 4*3*2*3/2 = 36
4 ways to pick what joe dislikes, 3 ways to think what bath dislikes, and 2 ways to blink what Emi dislikes. Now, 3 ways to pick the pair that likes the last song, BUT we have to divide by 2 because those same two also like another song, and we don't want to overcount. So 4*3*2*3/2 = 36
msinghal
2012-02-23 19:41:37
3*4C2*2*1=36
3*4C2*2*1=36
ss5188
2012-02-23 19:41:37
36
36
duketip10
2012-02-23 19:41:37
6*3*2
6*3*2
super_pi314
2012-02-23 19:41:37
36
36
.cpp
2012-02-23 19:41:37
36
36
christerson
2012-02-23 19:41:37
36
36
copeland
2012-02-23 19:41:51
What's the final answer?
What's the final answer?
AkshajK
2012-02-23 19:42:32
132
132
somanywindows
2012-02-23 19:42:32
132
132
tan90
2012-02-23 19:42:32
132
132
alex31415
2012-02-23 19:42:32
132
132
Seedleaf
2012-02-23 19:42:32
132 --> B
132 --> B
willwang123
2012-02-23 19:42:32
(B)
(B)
thepisong
2012-02-23 19:42:32
132
132
sparkles257
2012-02-23 19:42:32
add em up B
add em up B
va2010
2012-02-23 19:42:32
132 (B)
132 (B)
Polynomial
2012-02-23 19:42:32
132
132
pr0likethis
2012-02-23 19:42:32
132
132
joshxiong
2012-02-23 19:42:32
B. 132
B. 132
RotomPlasma
2012-02-23 19:42:32
24+72+36=132
24+72+36=132
Relativity1618
2012-02-23 19:42:32
B.132
B.132
Polynomial
2012-02-23 19:42:32
B 132
B 132
Draco
2012-02-23 19:42:32
132
132
KeepingItReal
2012-02-23 19:42:32
6*36=132 so B
6*36=132 so B
ss5188
2012-02-23 19:42:32
132
132
Jasonion
2012-02-23 19:42:32
132
132
copeland
2012-02-23 19:42:54
The answer is (B).
The answer is (B).
munugala
2012-02-23 19:43:02
I like the songs you picked!!!
I like the songs you picked!!!
copeland
2012-02-23 19:43:04
Then you must not be one of the girls.
Then you must not be one of the girls.
copeland
2012-02-23 19:43:35
Alright, the bug problem.
Alright, the bug problem.
copeland
2012-02-23 19:43:45
Prepare yourselves for some mad graphing skillz.
Prepare yourselves for some mad graphing skillz.
mathdate
2012-02-23 19:44:07
this problem really bugged me
this problem really bugged me
copeland
2012-02-23 19:44:09
:)
:)
copeland
2012-02-23 19:44:20
This problem seems a little daunting. Let's get our feet wet by solving an easier version.
This problem seems a little daunting. Let's get our feet wet by solving an easier version.
copeland
2012-02-23 19:45:25
(I un-reversed the backwards arrows.)
(I un-reversed the backwards arrows.)
copeland
2012-02-23 19:45:29
How can we solve this problem?
How can we solve this problem?
sammyMaX
2012-02-23 19:46:14
Well that was if you couldn't go backwards
Well that was if you couldn't go backwards
sammyMaX
2012-02-23 19:46:14
Multiplication!
Multiplication!
alex31415
2012-02-23 19:46:14
constructive counting
constructive counting
swimmer
2012-02-23 19:46:14
putting numbers at points
putting numbers at points
Garchomp293
2012-02-23 19:46:14
finding the amount of ways to get to each point, then adding them all up
finding the amount of ways to get to each point, then adding them all up
cuttlefishhorn
2012-02-23 19:46:14
Go point by point! How many ways there are to get to each segment, and add them up to figure out the next segment
Go point by point! How many ways there are to get to each segment, and add them up to figure out the next segment
mentalgenius
2012-02-23 19:46:14
write down numbers at every corner
write down numbers at every corner
zheng
2012-02-23 19:46:14
Figuring out the number of paths to each point
Figuring out the number of paths to each point
Seedleaf
2012-02-23 19:46:14
there's 2 ways to go for the first intersection, then 3 ways, then 4 ways, then 5... so on.
there's 2 ways to go for the first intersection, then 3 ways, then 4 ways, then 5... so on.
copeland
2012-02-23 19:46:22
Now the little bug can only cross these vertical lines one time each:
Now the little bug can only cross these vertical lines one time each:
copeland
2012-02-23 19:46:29
How many ways can the bug cross the first line?
How many ways can the bug cross the first line?
Macromiidae
2012-02-23 19:46:54
2
2
teethpaste
2012-02-23 19:46:54
2
2
c0mbusti0n1295
2012-02-23 19:46:54
dos
dos
moose97
2012-02-23 19:46:54
2
2
timoteomo3
2012-02-23 19:46:54
2
2
flyrain
2012-02-23 19:46:54
2.
2.
kenniky
2012-02-23 19:46:54
2
2
AkshajK
2012-02-23 19:46:54
2
2
va2010
2012-02-23 19:46:54
2
2
sammyMaX
2012-02-23 19:46:54
2
2
iamnew
2012-02-23 19:46:54
2
2
va2010
2012-02-23 19:46:54
2
2
zheng
2012-02-23 19:46:54
2
2
PChang
2012-02-23 19:46:54
2
2
pgmath
2012-02-23 19:46:54
2
2
177dum
2012-02-23 19:46:54
2
2
copeland
2012-02-23 19:46:58
2: either she goes through the top edge or the bottom edge.
2: either she goes through the top edge or the bottom edge.
copeland
2012-02-23 19:47:01
How many ways can the bug cross the first two lines?
How many ways can the bug cross the first two lines?
Garchomp293
2012-02-23 19:47:30
6
6
Jzou
2012-02-23 19:47:30
6
6
willwang123
2012-02-23 19:47:30
2x3=6
2x3=6
PChang
2012-02-23 19:47:30
6
6
timoteomo3
2012-02-23 19:47:30
2*3=6!
2*3=6!
pubado
2012-02-23 19:47:30
6
6
va2010
2012-02-23 19:47:30
2*3=6
2*3=6
ss5188
2012-02-23 19:47:30
6
6
trip
2012-02-23 19:47:30
6
6
martian179
2012-02-23 19:47:30
2 * 3 = 6
2 * 3 = 6
matthe44
2012-02-23 19:47:30
6
6
ptes77
2012-02-23 19:47:30
6
6
Mattchu386
2012-02-23 19:47:30
6
6
Draco
2012-02-23 19:47:30
6
6
copeland
2012-02-23 19:47:34
2*3. For either choice crossing the first line, there are three ways to cross the second line. Since the bug cannot pass along any edge more than once, there's exactly one path that uses one of the first edges and one of the second edges.
2*3. For either choice crossing the first line, there are three ways to cross the second line. Since the bug cannot pass along any edge more than once, there's exactly one path that uses one of the first edges and one of the second edges.
copeland
2012-02-23 19:47:36
How many ways can the bug cross the entire diagram?
How many ways can the bug cross the entire diagram?
iamnew
2012-02-23 19:48:23
2880
2880
RotomPlasma
2012-02-23 19:48:23
2*3*4*5*4*3*2=2880 ways
2*3*4*5*4*3*2=2880 ways
PChang
2012-02-23 19:48:23
2*3*4*5*4*3*2
2*3*4*5*4*3*2
willwang123
2012-02-23 19:48:23
2 x 3 x 4 x 5 x 4 x 3 x 2 = <some big number>
2 x 3 x 4 x 5 x 4 x 3 x 2 = <some big number>
InDe_eD
2012-02-23 19:48:23
2880
2880
davidkim2106
2012-02-23 19:48:23
2880??
2880??
Seedleaf
2012-02-23 19:48:23
2*3*4*5*4*3*2 = 2880
2*3*4*5*4*3*2 = 2880
Jzou
2012-02-23 19:48:23
2880
2880
teethpaste
2012-02-23 19:48:23
2*3*4*5*4*3*2=2880
2*3*4*5*4*3*2=2880
Macromiidae
2012-02-23 19:48:23
2*3*4*5*4*3*2=2880
2*3*4*5*4*3*2=2880
christerson
2012-02-23 19:48:23
2880
2880
MNL9082
2012-02-23 19:48:23
2*3*4*5*4*3*2
2*3*4*5*4*3*2
MathForFun
2012-02-23 19:48:23
2x3x4x5x4x3x2
2x3x4x5x4x3x2
eb8368
2012-02-23 19:48:23
2*3*4*5**4*3*2 = 2880
2*3*4*5**4*3*2 = 2880
copeland
2012-02-23 19:48:32
Therefore there are 2*3*4*5*4*3*2 paths that the bug can take through this maze.
Therefore there are 2*3*4*5*4*3*2 paths that the bug can take through this maze.
copeland
2012-02-23 19:48:45
Things get harder, though, when we look at the lattice with the reversed arrows. Our toy problem was more approachable because the bug could only cross each line once, but, more importantly, what the bug could do to cross one blue line did not depend on what the bug did to cross any previous blue lines.
Things get harder, though, when we look at the lattice with the reversed arrows. Our toy problem was more approachable because the bug could only cross each line once, but, more importantly, what the bug could do to cross one blue line did not depend on what the bug did to cross any previous blue lines.
copeland
2012-02-23 19:48:50
Let's go back to the original problem.
Let's go back to the original problem.
copeland
2012-02-23 19:49:17
Now the bug can cross many of those lines more than once and sometimes the ways the bug can cross depends on what the bug did in the past.
Now the bug can cross many of those lines more than once and sometimes the ways the bug can cross depends on what the bug did in the past.
copeland
2012-02-23 19:49:21
However it would still be nice if we could "factor" this problem. Do you see anything that we can draw?
However it would still be nice if we could "factor" this problem. Do you see anything that we can draw?
kenniky
2012-02-23 19:50:20
we can keep some of the lines from earlier
we can keep some of the lines from earlier
ksun48
2012-02-23 19:50:20
just the even vertical lines
just the even vertical lines
Jasmine8925
2012-02-23 19:50:20
draw vertical lines in the places where there are no backwards arrows
draw vertical lines in the places where there are no backwards arrows
briantix
2012-02-23 19:50:20
the lines from the previous problem escept the ones that cross a white arrow
the lines from the previous problem escept the ones that cross a white arrow
Polynomial
2012-02-23 19:50:20
Draw lines in the columns where there are no backward arrows
Draw lines in the columns where there are no backward arrows
christerson
2012-02-23 19:50:20
lines without backwards arrows?
lines without backwards arrows?
copeland
2012-02-23 19:50:24
We can still draw a couple of the blue lines:
We can still draw a couple of the blue lines:
copeland
2012-02-23 19:50:40
(There are a couple more we could draw, but, meh.)
(There are a couple more we could draw, but, meh.)
copeland
2012-02-23 19:50:46
Once the bug passes either of these blue lines its options do not depend on what happened before it hit the blue line (except possibly where it crossed the line).
Once the bug passes either of these blue lines its options do not depend on what happened before it hit the blue line (except possibly where it crossed the line).
copeland
2012-02-23 19:50:56
Let's talk about the left region. How are we going to count the ways that the bug can get from A to the left blue bar?
Let's talk about the left region. How are we going to count the ways that the bug can get from A to the left blue bar?
noedne
2012-02-23 19:51:50
casework
casework
fishinsea
2012-02-23 19:51:50
if it uses reverse or not
if it uses reverse or not
gurev
2012-02-23 19:51:50
the ways not going backwards, then ways going backwards
the ways not going backwards, then ways going backwards
Seedleaf
2012-02-23 19:51:50
with and without backwards arrow
with and without backwards arrow
RotomPlasma
2012-02-23 19:51:50
Only forwards and using the reverse to get to the other side
Only forwards and using the reverse to get to the other side
Jasmine8925
2012-02-23 19:51:50
two cases: bug goes backwards or doesn't go backwards
two cases: bug goes backwards or doesn't go backwards
martian179
2012-02-23 19:51:50
casework (whether u use the backwards arrow)?
casework (whether u use the backwards arrow)?
sammyMaX
2012-02-23 19:51:50
Casework: whether the bug goes backwards or not
Casework: whether the bug goes backwards or not
copeland
2012-02-23 19:51:56
We should split it into cases. Either the bug goes through the white arrow or it does not.
We should split it into cases. Either the bug goes through the white arrow or it does not.
copeland
2012-02-23 19:52:00
How many paths can the bug take without going through the white arrow?
How many paths can the bug take without going through the white arrow?
googol.plex
2012-02-23 19:52:31
16
16
Jzou
2012-02-23 19:52:31
16
16
qaryaendpetyp
2012-02-23 19:52:31
16
16
azure55
2012-02-23 19:52:34
2*2*4=16
2*2*4=16
qaryaendpetyp
2012-02-23 19:52:34
2*2*4=16
2*2*4=16
Iggy Iguana
2012-02-23 19:52:34
2*2*4=16
2*2*4=16
ksun48
2012-02-23 19:52:34
2*2*4=16
2*2*4=16
Seedleaf
2012-02-23 19:52:34
2*2*4 = 16
2*2*4 = 16
superpi83
2012-02-23 19:52:34
2*2*4=16
2*2*4=16
fishinsea
2012-02-23 19:52:34
2*2*4 = 16
2*2*4 = 16
AlcumusGuy
2012-02-23 19:52:34
16?
16?
lucylai
2012-02-23 19:52:34
16
16
copeland
2012-02-23 19:52:36
There are 16 of these paths: 2 choices for the first segment, 2 for the second and 4 for the third. (We might want to keep track, so notice that these choices are entirely symmetric. Four of these paths go through each of the four segments on the first blue line.)
There are 16 of these paths: 2 choices for the first segment, 2 for the second and 4 for the third. (We might want to keep track, so notice that these choices are entirely symmetric. Four of these paths go through each of the four segments on the first blue line.)
copeland
2012-02-23 19:52:45
Now how many paths cross the first white arrow on the way to the first blue line?
Now how many paths cross the first white arrow on the way to the first blue line?
PChang
2012-02-23 19:53:29
2*2*4
2*2*4
AkshajK
2012-02-23 19:53:29
4
4
superpi83
2012-02-23 19:53:29
2*2=4
2*2=4
mcdonalds106_7
2012-02-23 19:53:29
4
4
Macromiidae
2012-02-23 19:53:29
4?
4?
ksun48
2012-02-23 19:53:29
2*2=4
2*2=4
ifailedmathcounts
2012-02-23 19:53:29
4
4
MNL9082
2012-02-23 19:53:29
Four
Four
InDe_eD
2012-02-23 19:53:29
4
4
teethpaste
2012-02-23 19:53:29
4
4
Seedleaf
2012-02-23 19:53:29
4 ways
4 ways
copeland
2012-02-23 19:53:35
If a path crosses the top arrow first then it must start in this way:
If a path crosses the top arrow first then it must start in this way:
copeland
2012-02-23 19:53:43
There are only two choices from here to cross the middle line (one of the two bottom paths).
There are only two choices from here to cross the middle line (one of the two bottom paths).
copeland
2012-02-23 19:53:47
By symmetry, there are 2 choices for a path that crosses the bottom segment first.
By symmetry, there are 2 choices for a path that crosses the bottom segment first.
copeland
2012-02-23 19:53:50
There are a total of 4 paths. Each of these four paths cross a different arrow.
There are a total of 4 paths. Each of these four paths cross a different arrow.
copeland
2012-02-23 19:53:55
Now we take score. There are exactly 4+1=5 ways to get from point A to each of those four arrows along the first blue line.
Now we take score. There are exactly 4+1=5 ways to get from point A to each of those four arrows along the first blue line.
copeland
2012-02-23 19:54:47
What does this picture tell us about the original problem?
What does this picture tell us about the original problem?
Iggy Iguana
2012-02-23 19:55:28
so that means the answer is E because it is the only one that is divisible by 20
so that means the answer is E because it is the only one that is divisible by 20
gurev
2012-02-23 19:55:28
Now you know that it has to be divisible byfive right. So your done
Now you know that it has to be divisible byfive right. So your done
teethpaste
2012-02-23 19:55:28
it is a multiple of 5
it is a multiple of 5
kenniky
2012-02-23 19:55:28
It involves 5
It involves 5
Jasonion
2012-02-23 19:55:28
u don't have to do any more work, since E is the only multiple of 5?
u don't have to do any more work, since E is the only multiple of 5?
AkshajK
2012-02-23 19:55:28
its a mulitple of 5
its a mulitple of 5
superpi83
2012-02-23 19:55:28
the answer is a multiple of 5. WOW. so E.
the answer is a multiple of 5. WOW. so E.
Iggy Iguana
2012-02-23 19:55:28
number of paths is divible by 20
number of paths is divible by 20
somanywindows
2012-02-23 19:55:28
the answer has to be a multiple of 5
the answer has to be a multiple of 5
alex31415
2012-02-23 19:55:28
it must be a multiple of 5, so the answer must be 2400(E)?
it must be a multiple of 5, so the answer must be 2400(E)?
fortenforge
2012-02-23 19:55:28
The answer must be divisible by 5
The answer must be divisible by 5
copeland
2012-02-23 19:55:37
BLAAH!! How lame. :(
BLAAH!! How lame. :(
copeland
2012-02-23 19:55:43
No matter what happens the rest of the way, the number of paths is guaranteed to be a multiple of 5. The only solution given that is a multiple of 5 is the 2400, so that has to be the answer.
No matter what happens the rest of the way, the number of paths is guaranteed to be a multiple of 5. The only solution given that is a multiple of 5 is the 2400, so that has to be the answer.
copeland
2012-02-23 19:55:57
Good going AMC dudes. . . .
Good going AMC dudes. . . .
copeland
2012-02-23 19:55:58
Oh well. Let's see if this problem has anything more to teach us.
Oh well. Let's see if this problem has anything more to teach us.
copeland
2012-02-23 19:56:07
Can we learn anything more from what we've done so far?
Can we learn anything more from what we've done so far?
copeland
2012-02-23 19:57:10
My favorite thing in the mathiverse is symmetry.
My favorite thing in the mathiverse is symmetry.
copeland
2012-02-23 19:57:16
Do we have any symmetry here?
Do we have any symmetry here?
mentalgenius
2012-02-23 19:57:53
the last section of the graph is the same as the first one is
the last section of the graph is the same as the first one is
fractals
2012-02-23 19:57:53
Same for the other side, at the end
Same for the other side, at the end
ifailedmathcounts
2012-02-23 19:57:53
it's the same from teh 2nd line to B, path is reversible
it's the same from teh 2nd line to B, path is reversible
RotomPlasma
2012-02-23 19:57:53
Lololol. The entire field is symmetrical, so there are 5 solutions at the right, too.
Lololol. The entire field is symmetrical, so there are 5 solutions at the right, too.
azure55
2012-02-23 19:57:53
the rightmost part and the leftmost part we just calculated
the rightmost part and the leftmost part we just calculated
sammyMaX
2012-02-23 19:57:53
Yes, the first section and the third section are the same
Yes, the first section and the third section are the same
copeland
2012-02-23 19:58:00
There is symmetry! We also know that there are 5 ways to get from any of the segments along the right blue line to the point B.
There is symmetry! We also know that there are 5 ways to get from any of the segments along the right blue line to the point B.
copeland
2012-02-23 19:58:06
Now we just need to find all of the ways to get from any point on the first blue line to any point on the second blue line and multiply by 25.
Now we just need to find all of the ways to get from any point on the first blue line to any point on the second blue line and multiply by 25.
copeland
2012-02-23 19:58:17
How many paths pass from one blue line to the other and DO NOT use the white arrow?
How many paths pass from one blue line to the other and DO NOT use the white arrow?
copeland
2012-02-23 19:58:41
(Considering ALL entry and exit points.)
(Considering ALL entry and exit points.)
fractals
2012-02-23 19:59:02
4 * 4 * 4 = 64
4 * 4 * 4 = 64
ptes77
2012-02-23 19:59:02
64
64
RotomPlasma
2012-02-23 19:59:02
4^3=64
4^3=64
mcdonalds106_7
2012-02-23 19:59:02
64
64
Goos
2012-02-23 19:59:02
4^3=64
4^3=64
lucylai
2012-02-23 19:59:02
64
64
fishinsea
2012-02-23 19:59:02
64
64
christerson
2012-02-23 19:59:02
64
64
Iggy Iguana
2012-02-23 19:59:02
4*4=16
4*4=16
NeilOnnsu
2012-02-23 19:59:02
64
64
copeland
2012-02-23 19:59:05
There are 4 choices for entry, 4 choices for exit and 4 choices for the middle path so there are 64 total choices in this case.
There are 4 choices for entry, 4 choices for exit and 4 choices for the middle path so there are 64 total choices in this case.
copeland
2012-02-23 19:59:08
How many paths are there from one blue line to the other that pass through the white arrow?
How many paths are there from one blue line to the other that pass through the white arrow?
copeland
2012-02-23 20:00:05
This is more subtle: now the start and end paths are dependent. The best way to count is by considering whether the white path is entered from the top or the bottom. One case has paths that all use these edges:
This is more subtle: now the start and end paths are dependent. The best way to count is by considering whether the white path is entered from the top or the bottom. One case has paths that all use these edges:
copeland
2012-02-23 20:00:18
How many paths are there like that?
How many paths are there like that?
copeland
2012-02-23 20:01:02
How many paths through the middle section are there that contain that subpath, I mean?
How many paths through the middle section are there that contain that subpath, I mean?
AkshajK
2012-02-23 20:01:51
16
16
ifailedmathcounts
2012-02-23 20:01:51
16
16
superpi83
2012-02-23 20:01:51
16
16
Iggy Iguana
2012-02-23 20:01:51
2*2*2*2=16
2*2*2*2=16
mcdonalds106_7
2012-02-23 20:01:51
16
16
noedne
2012-02-23 20:01:51
2*2*2*2=16
2*2*2*2=16
canadian
2012-02-23 20:01:51
2^4=16
2^4=16
copeland
2012-02-23 20:02:06
There are 2^4=16 such paths. By symmetry there are 16 paths from blue to blue that go through the middle white arrow in the other direction as well.
There are 2^4=16 such paths. By symmetry there are 16 paths from blue to blue that go through the middle white arrow in the other direction as well.
copeland
2012-02-23 20:02:18
That gives a total of 96 paths through the middle section. How many total paths can the bug take?
That gives a total of 96 paths through the middle section. How many total paths can the bug take?
rlingineni
2012-02-23 20:02:54
2400
2400
noedne
2012-02-23 20:02:54
2400E
2400E
mcdonalds106_7
2012-02-23 20:02:54
25*96=2400 or E
25*96=2400 or E
sparkles257
2012-02-23 20:02:54
5*5*96=2400
5*5*96=2400
coldsummer
2012-02-23 20:02:54
2400
2400
iamnew
2012-02-23 20:02:54
2400
2400
Seedleaf
2012-02-23 20:02:54
96*25 = 2400 (E)
96*25 = 2400 (E)
NeilOnnsu
2012-02-23 20:02:54
2400
2400
negativebplusorminus
2012-02-23 20:02:54
2400
2400
fishinsea
2012-02-23 20:02:54
96*25=2400
96*25=2400
superpi83
2012-02-23 20:02:54
96*25=2400 E
96*25=2400 E
matemyday1234567890
2012-02-23 20:02:54
2400
2400
numberwiz
2012-02-23 20:02:54
2400
2400
BOGTRO
2012-02-23 20:02:54
25*96=2400
25*96=2400
noedne
2012-02-23 20:02:54
96*25=2400 so the answer is E
96*25=2400 so the answer is E
ringwraith10
2012-02-23 20:02:54
2400
2400
va2010
2012-02-23 20:02:54
96*5*5=2400
96*5*5=2400
martian179
2012-02-23 20:02:54
5 * 96 * 5 = 2400
5 * 96 * 5 = 2400
ringwraith10
2012-02-23 20:02:54
25*96
25*96
timoteomo3
2012-02-23 20:02:54
2400! (E)
2400! (E)
copeland
2012-02-23 20:03:01
The answer is (E).
The answer is (E).
copeland
2012-02-23 20:03:04
Super.
Super.
sammy1615
2012-02-23 20:03:19
is that all for amc 10?
is that all for amc 10?
copeland
2012-02-23 20:03:32
Yeah, now we'll take a short break and then move on to the AMC12.
Yeah, now we'll take a short break and then move on to the AMC12.
copeland
2012-02-23 20:03:49
It's 5:04 on my desk. Meet me back on my desk at 5:09.
It's 5:04 on my desk. Meet me back on my desk at 5:09.
rlingineni
2012-02-23 20:04:15
That took 1 hr, on the AMC we're only given 75 min
That took 1 hr, on the AMC we're only given 75 min
copeland
2012-02-23 20:04:17
But we solved that last one about 15min ago.
But we solved that last one about 15min ago.
BOGTRO
2012-02-23 20:05:03
we went through a lot more explanation than we would have on actual AMC yes?
we went through a lot more explanation than we would have on actual AMC yes?
copeland
2012-02-23 20:05:05
Yes. The AMC explanations consist of a few tiny bubbles.
Yes. The AMC explanations consist of a few tiny bubbles.
Mrdavid445
2012-02-23 20:06:44
If we finish early, can you go over AMC 12 problems before 20?
If we finish early, can you go over AMC 12 problems before 20?
copeland
2012-02-23 20:06:45
Yes, we may do that.
Yes, we may do that.
negativebplusorminus
2012-02-23 20:07:38
What do you think about number 12 of the AMC 12B, in terms of the interpretation? (there's a raging debate between D and E).
What do you think about number 12 of the AMC 12B, in terms of the interpretation? (there's a raging debate between D and E).
copeland
2012-02-23 20:07:42
The one with the sequences of ones and zeros? I'm on the side of not having ones still makes you such a sequence.
The one with the sequences of ones and zeros? I'm on the side of not having ones still makes you such a sequence.
copeland
2012-02-23 20:07:47
But I didn't think very hard about it.
But I didn't think very hard about it.
Klu2014
2012-02-23 20:08:11
When will this jam end? At what time
When will this jam end? At what time
zheng
2012-02-23 20:08:11
What time are we supposed to finish?
What time are we supposed to finish?
copeland
2012-02-23 20:08:13
Probably about an hour more? Depends on how quickly we move.
Probably about an hour more? Depends on how quickly we move.
copeland
2012-02-23 20:08:41
Breaktime's over! TIME FOR MORE MATH!
Breaktime's over! TIME FOR MORE MATH!
copeland
2012-02-23 20:08:45
Sorry. I get excited.
Sorry. I get excited.
Klu2014
2012-02-23 20:08:59
YAY!
YAY!
RotomPlasma
2012-02-23 20:08:59
YAY :D
YAY :D
copeland
2012-02-23 20:09:02
Looks like you two do, too.
Looks like you two do, too.
copeland
2012-02-23 20:09:27
This was a funny point on the exam. There's a chance that you came across this problem armed with WAY more tricks up your sleeve than is actually healthy, and spun your wheels with a lot of really sophisticated machines.
This was a funny point on the exam. There's a chance that you came across this problem armed with WAY more tricks up your sleeve than is actually healthy, and spun your wheels with a lot of really sophisticated machines.
copeland
2012-02-23 20:09:37
Hopefully not, though. Hopefully you remembered one of the most important rules: if you see a lot of equal angles you should look for. . . .
Hopefully not, though. Hopefully you remembered one of the most important rules: if you see a lot of equal angles you should look for. . . .
.cpp
2012-02-23 20:10:07
Similar triangles!
Similar triangles!
fishinsea
2012-02-23 20:10:07
similar triangles
similar triangles
negativebplusorminus
2012-02-23 20:10:07
Similar triangles.
Similar triangles.
iwantcombo
2012-02-23 20:10:07
similar triangles
similar triangles
mentalgenius
2012-02-23 20:10:07
similar + congruent triangles
similar + congruent triangles
tan90
2012-02-23 20:10:07
Similar triangles
Similar triangles
azure55
2012-02-23 20:10:07
similar trangles
similar trangles
mdecross
2012-02-23 20:10:07
similar triangles
similar triangles
canadian
2012-02-23 20:10:07
Similar triangles
Similar triangles
Iggy Iguana
2012-02-23 20:10:07
similar triangles
similar triangles
Mrdavid445
2012-02-23 20:10:07
similar stuff?
similar stuff?
zheng
2012-02-23 20:10:07
similar triangles
similar triangles
coldsummer
2012-02-23 20:10:07
similarity
similarity
copeland
2012-02-23 20:10:10
Similar triangles!
Similar triangles!
copeland
2012-02-23 20:10:19
Super, good job. We're almost about halfway home now.
Super, good job. We're almost about halfway home now.
copeland
2012-02-23 20:10:23
Do you see any similar triangles?
Do you see any similar triangles?
tan90
2012-02-23 20:11:28
ABX is congruent to YEZ
ABX is congruent to YEZ
himym83
2012-02-23 20:11:28
abx, zey
abx, zey
briantix
2012-02-23 20:11:28
ABX~YEZ
ABX~YEZ
Mrdavid445
2012-02-23 20:11:28
ABX and ZEY
ABX and ZEY
christerson
2012-02-23 20:11:28
ZEY and XBA
ZEY and XBA
TigerSneak1
2012-02-23 20:11:28
ABX and YEZ
ABX and YEZ
negativebplusorminus
2012-02-23 20:11:28
ABX and ZEY, perhaps?
ABX and ZEY, perhaps?
numberwiz
2012-02-23 20:11:28
abx and yez
abx and yez
copeland
2012-02-23 20:11:46
Also, always put the vertices in your similar triangles in the same order.
Also, always put the vertices in your similar triangles in the same order.
copeland
2012-02-23 20:11:55
This will keep you from making a LOT of mistakes.
This will keep you from making a LOT of mistakes.
copeland
2012-02-23 20:12:38
At this point in solving, I decided to spin my paper so that that bit was at the bottom.
At this point in solving, I decided to spin my paper so that that bit was at the bottom.
copeland
2012-02-23 20:12:46
And the bits we actually know things about are:
And the bits we actually know things about are:
copeland
2012-02-23 20:12:56
That's a little cleaner. Here's the actual data we know:
That's a little cleaner. Here's the actual data we know:
copeland
2012-02-23 20:13:15
Phew OK. We've written stuff down so we don't have to remember anymore.
Phew OK. We've written stuff down so we don't have to remember anymore.
copeland
2012-02-23 20:13:20
It's hard to think while remembering. . .
It's hard to think while remembering. . .
copeland
2012-02-23 20:13:24
That is nice, but we haven't really figured out how to use the information about the angles at E and F yet.
That is nice, but we haven't really figured out how to use the information about the angles at E and F yet.
copeland
2012-02-23 20:13:40
How are these angles related?
How are these angles related?
Klu2014
2012-02-23 20:14:06
120 degrees
120 degrees
mentalgenius
2012-02-23 20:14:06
angle E = angle F = 120 degrees
angle E = angle F = 120 degrees
numberwiz
2012-02-23 20:14:06
120
120
noedne
2012-02-23 20:14:06
congruent
congruent
negativebplusorminus
2012-02-23 20:14:06
They're equal.
They're equal.
Iggy Iguana
2012-02-23 20:14:06
they are equal, all 120 degrees
they are equal, all 120 degrees
TeamJacob
2012-02-23 20:14:06
120degrees
120degrees
RotomPlasma
2012-02-23 20:14:06
They're equal...?
They're equal...?
AkshajK
2012-02-23 20:14:06
they are both 120
they are both 120
fractals
2012-02-23 20:14:06
They both equal 120 degrees
They both equal 120 degrees
Seedleaf
2012-02-23 20:14:06
they are equal to 120
they are equal to 120
sammyMaX
2012-02-23 20:14:06
Congruent! (both 120)
Congruent! (both 120)
copeland
2012-02-23 20:14:11
They are both equal to 120 degrees. How do we use that?
They are both equal to 120 degrees. How do we use that?
alex31415
2012-02-23 20:14:58
law of cosines?
law of cosines?
negativebplusorminus
2012-02-23 20:14:58
Law of Cosines?
Law of Cosines?
numberwiz
2012-02-23 20:14:58
law of cos
law of cos
briantix
2012-02-23 20:14:58
law of cosines?
law of cosines?
tan90
2012-02-23 20:14:58
Law of cosines/sines?
Law of cosines/sines?
zheng
2012-02-23 20:15:00
cosine law?
cosine law?
Seedleaf
2012-02-23 20:15:00
law of cosines to find other side (YZ or ZA)
law of cosines to find other side (YZ or ZA)
mentalgenius
2012-02-23 20:15:00
law of cosines?
law of cosines?
.cpp
2012-02-23 20:15:00
Law of cosines.
Law of cosines.
copeland
2012-02-23 20:15:17
We could try some Law of Sines or Law of Cosines shenanigans and that will work, eventually, but a lot of times the Law of Cosines for 120 degrees just boils down to. . . .
We could try some Law of Sines or Law of Cosines shenanigans and that will work, eventually, but a lot of times the Law of Cosines for 120 degrees just boils down to. . . .
c0mbusti0n1295
2012-02-23 20:15:47
30 60 90 triangles?
30 60 90 triangles?
coldsummer
2012-02-23 20:15:47
create 30-60-90 triangles
create 30-60-90 triangles
number.sense
2012-02-23 20:15:47
30-60-90 triangles
30-60-90 triangles
xxrxxhxx
2012-02-23 20:15:47
30-60-90 triangles
30-60-90 triangles
negativebplusorminus
2012-02-23 20:15:47
Pythagoras?
Pythagoras?
copeland
2012-02-23 20:15:59
Good. Where are they?
Good. Where are they?
tan90
2012-02-23 20:16:41
Draw altitudes.
Draw altitudes.
RotomPlasma
2012-02-23 20:16:41
Draw altitudes
Draw altitudes
fractals
2012-02-23 20:16:41
Drop perpendiculars
Drop perpendiculars
c0mbusti0n1295
2012-02-23 20:16:41
lol i was thinking of drawing altitudes..
lol i was thinking of drawing altitudes..
mcdonalds106_7
2012-02-23 20:16:41
draw the altitudes
draw the altitudes
KeepingItReal
2012-02-23 20:16:41
draw right angles
draw right angles
copeland
2012-02-23 20:16:42
Super, where?
Super, where?
copeland
2012-02-23 20:17:42
Notice that what we want to find is a 60 degree angle, remember?
Notice that what we want to find is a 60 degree angle, remember?
fractals
2012-02-23 20:18:13
A and Y to EF
A and Y to EF
mdecross
2012-02-23 20:18:13
from A and Y
from A and Y
mentalgenius
2012-02-23 20:18:13
with hypotenuses YE and AF, with perpendiculars vertical
with hypotenuses YE and AF, with perpendiculars vertical
TeamJacob
2012-02-23 20:18:13
draw vertical lines from y and a
draw vertical lines from y and a
proglote
2012-02-23 20:18:13
Y to EF
Y to EF
silvernight
2012-02-23 20:18:13
extend EF and draw a perpedicular containing A to line EF
extend EF and draw a perpedicular containing A to line EF
copeland
2012-02-23 20:18:17
If we drop perpendiculars from Y and from A to the line through E and F we get a couple of nice 30-60-90 triangles.
If we drop perpendiculars from Y and from A to the line through E and F we get a couple of nice 30-60-90 triangles.
copeland
2012-02-23 20:18:20
(The picture will be a little easier if we ignore the top of the diagram entirely for a while.)
(The picture will be a little easier if we ignore the top of the diagram entirely for a while.)
copeland
2012-02-23 20:18:33
Now what distances do we know?
Now what distances do we know?
fractals
2012-02-23 20:19:12
LE = 20, YL = 20sqrt3
LE = 20, YL = 20sqrt3
alex31415
2012-02-23 20:19:12
YL=20sqrt3, LE=20
YL=20sqrt3, LE=20
PChang
2012-02-23 20:19:12
EL,LY
EL,LY
TigerSneak1
2012-02-23 20:19:12
LE and LY via 30-60-90
LE and LY via 30-60-90
somanywindows
2012-02-23 20:19:12
LE = 20, YL = 20sqrt3
LE = 20, YL = 20sqrt3
proglote
2012-02-23 20:19:12
YL and EL
YL and EL
Smart Pi
2012-02-23 20:19:12
YL and LE?
YL and LE?
copeland
2012-02-23 20:19:20
Since YEL is a 30-60-90 triangle with hypotenuse 40 we know both of the legs:
Since YEL is a 30-60-90 triangle with hypotenuse 40 we know both of the legs:
copeland
2012-02-23 20:20:21
What's left that we haven't incoroporated into the picture?
What's left that we haven't incoroporated into the picture?
fractals
2012-02-23 20:20:35
angle YZA = 90
angle YZA = 90
number.sense
2012-02-23 20:20:35
YZ = AZ and its a right angle
YZ = AZ and its a right angle
pjyoung
2012-02-23 20:20:35
YZ and AZ?
YZ and AZ?
proglote
2012-02-23 20:20:35
the fact that YZ = AZ (sides of square)
the fact that YZ = AZ (sides of square)
mentalgenius
2012-02-23 20:20:35
YZAX is a square
YZAX is a square
.cpp
2012-02-23 20:20:35
YZ=AZ.
YZ=AZ.
Yongyi781
2012-02-23 20:20:35
AZY is a right angle!
AZY is a right angle!
copeland
2012-02-23 20:20:45
We haven't used that AXYZ is a square. What does that tell us?
We haven't used that AXYZ is a square. What does that tell us?
copeland
2012-02-23 20:20:52
(It tells us 2 things.)
(It tells us 2 things.)
pgmath
2012-02-23 20:22:35
YZL is similar to ZAR and YZ = AZ
YZL is similar to ZAR and YZ = AZ
canadian
2012-02-23 20:22:35
Triangles YLZ and ZRA are similar
Triangles YLZ and ZRA are similar
Iggy Iguana
2012-02-23 20:22:35
congruent triangles lyz, rza
congruent triangles lyz, rza
zheng
2012-02-23 20:22:35
YLZ and ZRA are congruent since YZ=AZ and their angles are equal
YLZ and ZRA are congruent since YZ=AZ and their angles are equal
Yongyi781
2012-02-23 20:22:35
YLZ is congruent to ZRA
YLZ is congruent to ZRA
mdecross
2012-02-23 20:22:35
also that YZ and AZ are equal in length
also that YZ and AZ are equal in length
copeland
2012-02-23 20:22:42
Since AXYZ is a square, AZY is a right angle. That forces YZL and AZR to be complementary angles.
Since AXYZ is a square, AZY is a right angle. That forces YZL and AZR to be complementary angles.
copeland
2012-02-23 20:22:46
By AA similarity, YZL and ZAR are similar. Furthermore, since YZ=ZA these are actually congruent!
By AA similarity, YZL and ZAR are similar. Furthermore, since YZ=ZA these are actually congruent!
copeland
2012-02-23 20:22:50
Therefore we can copy the distance YL=ZR.
Therefore we can copy the distance YL=ZR.
copeland
2012-02-23 20:23:31
Our diagram's looking good, but now we've run out of information. This is to be expected: we should have to solve SOMETHING to figure out where A is. In particular, we still need to find the scale of the triangle AFR. So let's go ahead an bite the bullet and throw a variable in there.
Our diagram's looking good, but now we've run out of information. This is to be expected: we should have to solve SOMETHING to figure out where A is. In particular, we still need to find the scale of the triangle AFR. So let's go ahead an bite the bullet and throw a variable in there.
copeland
2012-02-23 20:23:42
Here's a nice thing: waiting to introduce the variable allows us to pick one that will lead to the least headache when solving.
Here's a nice thing: waiting to introduce the variable allows us to pick one that will lead to the least headache when solving.
copeland
2012-02-23 20:24:55
Here's the philosophy. We're trying to lock down this triangle on the right. That's where the variable is going to be used. What edge should we make the variable?
Here's the philosophy. We're trying to lock down this triangle on the right. That's where the variable is going to be used. What edge should we make the variable?
coldsummer
2012-02-23 20:25:22
FR
FR
zheng
2012-02-23 20:25:22
FR
FR
KeepingItReal
2012-02-23 20:25:22
FR
FR
Yongyi781
2012-02-23 20:25:22
FR?
FR?
Iggy Iguana
2012-02-23 20:25:22
fr?
fr?
adamdai97
2012-02-23 20:25:22
FR
FR
proglote
2012-02-23 20:25:22
FR
FR
copeland
2012-02-23 20:26:01
Now we can find x! I'll give you a moment.
Now we can find x! I'll give you a moment.
number.sense
2012-02-23 20:27:18
21
21
Goos
2012-02-23 20:27:18
x=21
x=21
sammyMaX
2012-02-23 20:27:18
21
21
christerson
2012-02-23 20:27:18
21
21
fractals
2012-02-23 20:27:18
xsqrt3 + 20sqrt3 = 20 + 41(sqrt3 - 1) + x, so x(sqrt3 - 1) = (41 - 20)(sqrt3 - 1) = 21(sqrt3 - 1), so x = 21.
xsqrt3 + 20sqrt3 = 20 + 41(sqrt3 - 1) + x, so x(sqrt3 - 1) = (41 - 20)(sqrt3 - 1) = 21(sqrt3 - 1), so x = 21.
mentalgenius
2012-02-23 20:27:18
x = 21
x = 21
Jzou
2012-02-23 20:27:18
x=21
x=21
Yongyi781
2012-02-23 20:27:18
x = 21!!!!!
x = 21!!!!!
copeland
2012-02-23 20:27:46
And what is the side-length of the square?
And what is the side-length of the square?
nikeballa96
2012-02-23 20:28:34
29sqrt3
29sqrt3
noedne
2012-02-23 20:28:34
A
A
fractals
2012-02-23 20:28:34
29sqrt3
29sqrt3
number.sense
2012-02-23 20:28:34
A
A
Seedleaf
2012-02-23 20:28:34
29sqrt3 -->A!
29sqrt3 -->A!
mentalgenius
2012-02-23 20:28:34
(A) 29 sqrt(3)
(A) 29 sqrt(3)
lucylai
2012-02-23 20:28:34
29sqrt3
29sqrt3
noedne
2012-02-23 20:28:34
20-21-29 *sqrt 3
20-21-29 *sqrt 3
lucylai
2012-02-23 20:28:34
29sqrt3 (A)
29sqrt3 (A)
christerson
2012-02-23 20:28:34
29 sqrt(3)
29 sqrt(3)
ksun48
2012-02-23 20:28:34
29sqrt{3}
29sqrt{3}
.cpp
2012-02-23 20:28:34
29sqrt{3}
29sqrt{3}
alex31415
2012-02-23 20:28:34
29sqrt2
29sqrt2
KeepingItReal
2012-02-23 20:28:34
by pythag, length is 29sqrt(3) so answer is A
by pythag, length is 29sqrt(3) so answer is A
copeland
2012-02-23 20:29:11
The answer is (A).
The answer is (A).
copeland
2012-02-23 20:29:33
Problem 22 on the 12B is the same as 25 on the 10B so we already solved that one.
Problem 22 on the 12B is the same as 25 on the 10B so we already solved that one.
copeland
2012-02-23 20:29:52
(They changed the answer choices from the original version on this problem. These were the updated options.)
(They changed the answer choices from the original version on this problem. These were the updated options.)
copeland
2012-02-23 20:30:05
I'll come clean with you on this one: the good way to do this problem is by writing all the answers down and calling yourself done. Using math on this problem turns out not to be so helpful unless you use a LOT of math.
I'll come clean with you on this one: the good way to do this problem is by writing all the answers down and calling yourself done. Using math on this problem turns out not to be so helpful unless you use a LOT of math.
copeland
2012-02-23 20:30:18
Let's try that. What z satisfy |z|=1 and are easy to work with?
Let's try that. What z satisfy |z|=1 and are easy to work with?
c0mbusti0n1295
2012-02-23 20:30:43
1, -1
1, -1
m1sterzer0
2012-02-23 20:30:43
-1
-1
einstein3.14
2012-02-23 20:30:43
-1,1
-1,1
somanywindows
2012-02-23 20:30:43
-1, 1
-1, 1
matemyday1234567890
2012-02-23 20:30:43
1
1
coldsummer
2012-02-23 20:30:43
1
1
KeepingItReal
2012-02-23 20:30:43
(1,0)
(1,0)
Animyth
2012-02-23 20:30:43
+or-1
+or-1
knowmath
2012-02-23 20:30:43
-1,1
-1,1
christerson
2012-02-23 20:30:43
1, -1
1, -1
TigerSneak1
2012-02-23 20:30:43
1?
1?
copeland
2012-02-23 20:30:47
First we have z=1.
First we have z=1.
copeland
2012-02-23 20:30:50
How many choices for a, b, c, and d give P(1)=0?
How many choices for a, b, c, and d give P(1)=0?
canadian
2012-02-23 20:31:26
non
non
fractals
2012-02-23 20:31:26
None
None
GoldenFrog1618
2012-02-23 20:31:26
it does not matter, it will give 0 to the sum
it does not matter, it will give 0 to the sum
alex31415
2012-02-23 20:31:26
it doesnt matter
it doesnt matter
matemyday1234567890
2012-02-23 20:31:26
0
0
soccerfan
2012-02-23 20:31:26
0
0
alex31415
2012-02-23 20:31:26
it will always be 0
it will always be 0
lucylai
2012-02-23 20:31:26
none
none
mentalgenius
2012-02-23 20:31:26
none
none
RotomPlasma
2012-02-23 20:31:26
0
0
mentalgenius
2012-02-23 20:31:26
ZERO
ZERO
copeland
2012-02-23 20:31:38
(Furthermore that doesn't contribute to the sum anyway.)
(Furthermore that doesn't contribute to the sum anyway.)
copeland
2012-02-23 20:31:43
The next simplest is z=-1. What polynomials of the form above satisfy P(-1)=0?
The next simplest is z=-1. What polynomials of the form above satisfy P(-1)=0?
knowmath
2012-02-23 20:32:20
a lot
a lot
iamnew
2012-02-23 20:32:20
-a+b-c+d=0
-a+b-c+d=0
lucylai
2012-02-23 20:32:20
4-a+b-c+d=0
4-a+b-c+d=0
fractals
2012-02-23 20:32:20
4 - a + b - c + d = 0
4 - a + b - c + d = 0
alex31415
2012-02-23 20:32:20
b+d+4=a+c
b+d+4=a+c
.cpp
2012-02-23 20:32:20
a+c = 4+b+d
a+c = 4+b+d
copeland
2012-02-23 20:32:34
Good, and how does the nondecreasing assumption narrow this?
Good, and how does the nondecreasing assumption narrow this?
copeland
2012-02-23 20:33:07
("Nonincreasing," I guess? Their inequality is written d to a.)
("Nonincreasing," I guess? Their inequality is written d to a.)
copeland
2012-02-23 20:33:59
Here are some observations:
Here are some observations:
m1sterzer0
2012-02-23 20:34:01
4=a, b=c, d=0
4=a, b=c, d=0
KeepingItReal
2012-02-23 20:34:01
or 1,1,3,3
or 1,1,3,3
.cpp
2012-02-23 20:34:01
a= 4, b = 2, c=2, d=0; a=4, b=1, c=1, d=0
a= 4, b = 2, c=2, d=0; a=4, b=1, c=1, d=0
pr0likethis
2012-02-23 20:34:01
a must be four, because if a was smaller than b>=c>=d wouldnt be true
a must be four, because if a was smaller than b>=c>=d wouldnt be true
copeland
2012-02-23 20:34:19
Can we make that formal?
Can we make that formal?
.cpp
2012-02-23 20:34:56
4>=a; b>=c, so 4+b>=a+c. Hence, 4=a, b=c, d=0.
4>=a; b>=c, so 4+b>=a+c. Hence, 4=a, b=c, d=0.
KeepingItReal
2012-02-23 20:35:56
8+2b
8+2b
Seedleaf
2012-02-23 20:35:56
8+2b
8+2b
superwatchdog
2012-02-23 20:35:56
8+2b
8+2b
iamnew
2012-02-23 20:35:56
8+2b
8+2b
adamdai97
2012-02-23 20:35:56
8+2b
8+2b
RotomPlasma
2012-02-23 20:35:56
8+2b
8+2b
TigerSneak1
2012-02-23 20:35:56
8 + 2b
8 + 2b
PChang
2012-02-23 20:35:56
8+2b
8+2b
copeland
2012-02-23 20:35:58
And as b varies from 0 to 4, what is the total sum?
And as b varies from 0 to 4, what is the total sum?
.cpp
2012-02-23 20:36:34
(8+2b) for b in range 0,4 = 40 + 20 = 60.
(8+2b) for b in range 0,4 = 40 + 20 = 60.
fractals
2012-02-23 20:36:34
40 + 2(1 + 2 + 3 + 4) = 60
40 + 2(1 + 2 + 3 + 4) = 60
Iggy Iguana
2012-02-23 20:36:34
60
60
pr0likethis
2012-02-23 20:36:34
8+10+12+14+16=60
8+10+12+14+16=60
number.sense
2012-02-23 20:36:34
60
60
Yongyi781
2012-02-23 20:36:34
60
60
azure55
2012-02-23 20:36:34
60
60
somanywindows
2012-02-23 20:36:34
60
60
iamnew
2012-02-23 20:36:34
60
60
pgmath
2012-02-23 20:36:34
60
60
super_pi314
2012-02-23 20:36:34
60
60
copeland
2012-02-23 20:36:57
What other z with |z|=1 might be good to try?
What other z with |z|=1 might be good to try?
Yongyi781
2012-02-23 20:38:16
kth roots of unity for k=3, 4... and 5!!! (and probably 6, 7, 8, and 9). And also non-roots of unity
kth roots of unity for k=3, 4... and 5!!! (and probably 6, 7, 8, and 9). And also non-roots of unity
plokclop
2012-02-23 20:38:16
The cube roots of 1.
The cube roots of 1.
RotomPlasma
2012-02-23 20:38:16
+/- i?
+/- i?
number.sense
2012-02-23 20:38:16
cis45
cis45
azure55
2012-02-23 20:38:16
cos45+isin45
cos45+isin45
ksun48
2012-02-23 20:38:16
cube rts of unity
cube rts of unity
pr0likethis
2012-02-23 20:38:16
in other words, cis(30, 45, 60, etc)
in other words, cis(30, 45, 60, etc)
copeland
2012-02-23 20:38:22
Let's look for other roots of unity. Do you see any more that work?
Let's look for other roots of unity. Do you see any more that work?
fractals
2012-02-23 20:39:15
cis(2pi/5)
cis(2pi/5)
ksun48
2012-02-23 20:39:15
5th roots?
5th roots?
copeland
2012-02-23 20:39:26
What polynomial does a primitive fifth root solve?
What polynomial does a primitive fifth root solve?
m1sterzer0
2012-02-23 20:40:03
5th roots work for (z^5-1)/(z-1)
5th roots work for (z^5-1)/(z-1)
Yongyi781
2012-02-23 20:40:03
x^4 + x^3 + x^2 + x + 1 = 0
x^4 + x^3 + x^2 + x + 1 = 0
lucylai
2012-02-23 20:40:03
x^5-1=0
x^5-1=0
robinpark
2012-02-23 20:40:03
z^5 - 1 = 0
z^5 - 1 = 0
.cpp
2012-02-23 20:40:03
x^4+x^3+x^2+x+1 = 0.
x^4+x^3+x^2+x+1 = 0.
fractals
2012-02-23 20:40:03
z^5 - 1 = 0, OR z^4 + z^3 + z^2 + z + 1 = 0 (it is primitive)
z^5 - 1 = 0, OR z^4 + z^3 + z^2 + z + 1 = 0 (it is primitive)
number.sense
2012-02-23 20:40:03
x^4 + x^3 + x^2 + x + 1
x^4 + x^3 + x^2 + x + 1
copeland
2012-02-23 20:40:12
That goes on the list.
That goes on the list.
copeland
2012-02-23 20:40:20
Any other roots of unity in sight?
Any other roots of unity in sight?
.cpp
2012-02-23 20:41:20
Cube roots work (e.g. 4x^2(x^2+x+1)).
Cube roots work (e.g. 4x^2(x^2+x+1)).
dinoboy
2012-02-23 20:41:20
the third and fourth roots of unity generate meaningful polynomials
the third and fourth roots of unity generate meaningful polynomials
knowmath
2012-02-23 20:41:20
fourth root
fourth root
copeland
2012-02-23 20:41:36
These two add 12+20=32 to our sum. That gives a running total of 92.
These two add 12+20=32 to our sum. That gives a running total of 92.
copeland
2012-02-23 20:41:42
No other roots of unity will work because the decreasing coefficients constraint forces our roots of unity equations to have consecutive powers of 4x^n.
No other roots of unity will work because the decreasing coefficients constraint forces our roots of unity equations to have consecutive powers of 4x^n.
copeland
2012-02-23 20:41:49
At this point I strongly suggest thinking a little more about how to make other polynomials, and in particular polynomials that will get us from a sum of 92 to one of the other values that are possible answers. Then bubble in 92 and go on.
At this point I strongly suggest thinking a little more about how to make other polynomials, and in particular polynomials that will get us from a sum of 92 to one of the other values that are possible answers. Then bubble in 92 and go on.
copeland
2012-02-23 20:42:11
However we can do a little better than that. I'll run through a nice way to deal with this next, but a little more quickly than we've been going so far.
However we can do a little better than that. I'll run through a nice way to deal with this next, but a little more quickly than we've been going so far.
ahaanomegas
2012-02-23 20:42:51
(B)
(B)
coldsummer
2012-02-23 20:42:51
so the answer is B?
so the answer is B?
superwatchdog
2012-02-23 20:42:51
B
B
copeland
2012-02-23 20:42:54
We don't KNOW that yet. At this point I hope you picked B and didn't spend any more time on this problem. We haven't shown anything except that the answer is at least 92.
We don't KNOW that yet. At this point I hope you picked B and didn't spend any more time on this problem. We haven't shown anything except that the answer is at least 92.
copeland
2012-02-23 20:43:11
Notice that all of the approaches we took work with roots of unity.
Notice that all of the approaches we took work with roots of unity.
copeland
2012-02-23 20:43:24
There's slightly better motivation for this.
There's slightly better motivation for this.
copeland
2012-02-23 20:43:37
A generating function approach allows us to combine all this data.
A generating function approach allows us to combine all this data.
copeland
2012-02-23 20:43:55
The coefficients of the bottom equation are always at least as large as the coefficients of the top equation, except for the leading 4z^5.
The coefficients of the bottom equation are always at least as large as the coefficients of the top equation, except for the leading 4z^5.
copeland
2012-02-23 20:44:35
Now notice that if P(z)=0 then this new function is also 0. However this new function also has a zero at z=1. Since the coefficients of P are nonnegative, P(1) is at least 4, so 1 cannot be a root of P.
Now notice that if P(z)=0 then this new function is also 0. However this new function also has a zero at z=1. Since the coefficients of P are nonnegative, P(1) is at least 4, so 1 cannot be a root of P.
copeland
2012-02-23 20:44:49
Therefore we can instead find polynomials of the form (z-1)P(z) for which there is a root of modulus 1 other than z=1.
Therefore we can instead find polynomials of the form (z-1)P(z) for which there is a root of modulus 1 other than z=1.
copeland
2012-02-23 20:44:53
What can we say about each of the coefficients of (z-1)P(z)?
What can we say about each of the coefficients of (z-1)P(z)?
fractals
2012-02-23 20:45:49
Negative except for 4z^5
Negative except for 4z^5
.cpp
2012-02-23 20:45:49
All negative except for the 4x^5 term.
All negative except for the 4x^5 term.
Iggy Iguana
2012-02-23 20:45:49
nonpositive
nonpositive
ksun48
2012-02-23 20:45:49
nonpositive, except for leading term
nonpositive, except for leading term
number.sense
2012-02-23 20:45:49
sign of constant term is reversed
sign of constant term is reversed
mdecross
2012-02-23 20:45:49
They are <= 0, except the leading coefficient
They are <= 0, except the leading coefficient
mentalgenius
2012-02-23 20:45:49
it is negative
it is negative
copeland
2012-02-23 20:45:56
What else can we say?
What else can we say?
copeland
2012-02-23 20:46:22
We have the numbers 4, (a-4), (b-a), (c-b), (d-c), and -d.
We have the numbers 4, (a-4), (b-a), (c-b), (d-c), and -d.
Goos
2012-02-23 20:47:25
sum of all coefficients is 0
sum of all coefficients is 0
Iggy Iguana
2012-02-23 20:47:25
sum is 0
sum is 0
sammyMaX
2012-02-23 20:47:25
They add to zero
They add to zero
.cpp
2012-02-23 20:47:25
Sum = 0.
Sum = 0.
mcdonalds106_7
2012-02-23 20:47:25
they sum to 0
they sum to 0
coldsummer
2012-02-23 20:47:25
the sum is 0
the sum is 0
superwatchdog
2012-02-23 20:47:25
the sum of the coefficients is zero
the sum of the coefficients is zero
mentalgenius
2012-02-23 20:47:25
Adding them gives zero?
Adding them gives zero?
copeland
2012-02-23 20:47:37
The leading coefficient is 4 (which is big) and all the remaining coefficients add to -4.
The leading coefficient is 4 (which is big) and all the remaining coefficients add to -4.
copeland
2012-02-23 20:47:45
If z is of modulus 1 then all of the terms below the leading term have to work together to cancel with the leading 4 and make 0.
If z is of modulus 1 then all of the terms below the leading term have to work together to cancel with the leading 4 and make 0.
copeland
2012-02-23 20:48:06
Now when comparing the sizes of elements, the best thing to try first is the triangle inequality.
Now when comparing the sizes of elements, the best thing to try first is the triangle inequality.
copeland
2012-02-23 20:48:39
That thing on the right is equal to 4, though, so we have equality.
That thing on the right is equal to 4, though, so we have equality.
copeland
2012-02-23 20:48:42
However when is the triangle inequality an actual equality?
However when is the triangle inequality an actual equality?
Seedleaf
2012-02-23 20:49:37
when its in a straight line
when its in a straight line
fractals
2012-02-23 20:49:37
Everything is on a line through the origin
Everything is on a line through the origin
Yongyi781
2012-02-23 20:49:37
When the complex numbers are real multiples of each other
When the complex numbers are real multiples of each other
timoteomo3
2012-02-23 20:49:37
when its a line!
when its a line!
distortedwalrus
2012-02-23 20:49:37
when the points are collinear
when the points are collinear
number.sense
2012-02-23 20:49:37
when they all form a straight line
when they all form a straight line
mentalgenius
2012-02-23 20:49:37
when there is a straight line
when there is a straight line
MathForFun
2012-02-23 20:49:37
When its a line... or degenerate
When its a line... or degenerate
copeland
2012-02-23 20:50:15
Continuing down this road tells us that we have to have z be SOME root of unity or have all of the coefficients equal to zero. However if all the coefficients are zero that contradicts that the sum of the coefficients is 4. So the only possible values for z are roots of unity and the answer we gave (with a little more work to rule out polynomials like 4x^4+4) does count all of the possible sums.
Continuing down this road tells us that we have to have z be SOME root of unity or have all of the coefficients equal to zero. However if all the coefficients are zero that contradicts that the sum of the coefficients is 4. So the only possible values for z are roots of unity and the answer we gave (with a little more work to rule out polynomials like 4x^4+4) does count all of the possible sums.
Yongyi781
2012-02-23 20:51:03
4x^4 + 4 doesn't have nonincreasing coefficients
4x^4 + 4 doesn't have nonincreasing coefficients
copeland
2012-02-23 20:51:15
Exactly.
Exactly.
copeland
2012-02-23 20:51:27
That's not precluded from the triangle inequality argument we gave, though.
That's not precluded from the triangle inequality argument we gave, though.
copeland
2012-02-23 20:51:41
Polynomials like that will pop out and you have to rule them out by hand.
Polynomials like that will pop out and you have to rule them out by hand.
pr0likethis
2012-02-23 20:51:50
Is it worth noting that 1, -1, i, and -i are 4th roots of unity, so all of our solutions were roots of unity up to 5?
Is it worth noting that 1, -1, i, and -i are 4th roots of unity, so all of our solutions were roots of unity up to 5?
number.sense
2012-02-23 20:51:50
wait so the answer is B?
wait so the answer is B?
copeland
2012-02-23 20:51:56
Yes!
Yes!
copeland
2012-02-23 20:52:02
Now. I have no idea how you were expected to do any of that last bit on the exam.
Now. I have no idea how you were expected to do any of that last bit on the exam.
copeland
2012-02-23 20:52:42
I haven't seen or found any argument that is even reasonably close to AMC12 level to get from what we did at the beginning to an honest proof that 92 is correct.
I haven't seen or found any argument that is even reasonably close to AMC12 level to get from what we did at the beginning to an honest proof that 92 is correct.
matemyday1234567890
2012-02-23 20:52:53
i would just guess b and move on instead of doing all the math after you said to bubble b
i would just guess b and move on instead of doing all the math after you said to bubble b
copeland
2012-02-23 20:52:56
Good!
Good!
copeland
2012-02-23 20:52:59
Always take my advice.
Always take my advice.
copeland
2012-02-23 20:53:02
Except when it's bad.
Except when it's bad.
copeland
2012-02-23 20:53:06
Now it's time to bubble and move on, though.
Now it's time to bubble and move on, though.
copeland
2012-02-23 20:53:27
Where should we start?
Where should we start?
.cpp
2012-02-23 20:54:15
Try some small values first.
Try some small values first.
nikeballa96
2012-02-23 20:54:15
try some examples...
try some examples...
copeland
2012-02-23 20:54:18
It couldn't hurt to draw a table of the first few, just to see what's happening. Here's my table.
It couldn't hurt to draw a table of the first few, just to see what's happening. Here's my table.
copeland
2012-02-23 20:54:43
If you're having trouble with a concept like "unbounded" when you hit a problem like this, if you have time play with it for a while and maybe the definition will clear itself up.
If you're having trouble with a concept like "unbounded" when you hit a problem like this, if you have time play with it for a while and maybe the definition will clear itself up.
copeland
2012-02-23 20:54:50
That will happen for us now.
That will happen for us now.
copeland
2012-02-23 20:54:58
Hmm. It looks like we are getting a lot of ones. Let's make a wider table and go down to f_2.
Hmm. It looks like we are getting a lot of ones. Let's make a wider table and go down to f_2.
copeland
2012-02-23 20:55:24
Huh. That's easy enough to extend since we already have f for these outputs.
Huh. That's easy enough to extend since we already have f for these outputs.
copeland
2012-02-23 20:55:30
We care about the vertical sequences. Are any of these sequences unbounded?
We care about the vertical sequences. Are any of these sequences unbounded?
.cpp
2012-02-23 20:55:52
16-27-16-27-... loops but is not unbounded.
16-27-16-27-... loops but is not unbounded.
ksun48
2012-02-23 20:55:52
nope!
nope!
canadian
2012-02-23 20:55:52
no
no
2718euler
2012-02-23 20:55:52
No
No
fractals
2012-02-23 20:55:52
None at all.
None at all.
Iggy Iguana
2012-02-23 20:55:52
no
no
number.sense
2012-02-23 20:55:52
not yet...
not yet...
Goos
2012-02-23 20:55:52
nope
nope
copeland
2012-02-23 20:55:55
No. All of them go to 1 except 16 which oscillates between 16 and 27.
No. All of them go to 1 except 16 which oscillates between 16 and 27.
copeland
2012-02-23 20:56:07
"Unbounded" means that the sequence gets bigger and bigger and bigger.
"Unbounded" means that the sequence gets bigger and bigger and bigger.
copeland
2012-02-23 20:56:22
16-27-16... never gets bigger than 27.
16-27-16... never gets bigger than 27.
copeland
2012-02-23 20:56:28
Why do the others disappear?
Why do the others disappear?
matemyday1234567890
2012-02-23 20:57:12
it changes from one only when the number is a multiple of a square number
it changes from one only when the number is a multiple of a square number
KeepingItReal
2012-02-23 20:57:12
no squares/cubes/etc
no squares/cubes/etc
canadian
2012-02-23 20:57:12
All the e's eventually become 0
All the e's eventually become 0
.cpp
2012-02-23 20:57:12
They slowly lose their exponential powers and become ordinary primes and then 1.
They slowly lose their exponential powers and become ordinary primes and then 1.
copeland
2012-02-23 20:57:19
Let's get a couple more examples on the table before we can think about this. What is the next sequence you're itching to try?
Let's get a couple more examples on the table before we can think about this. What is the next sequence you're itching to try?
.cpp
2012-02-23 20:58:07
f(32)
f(32)
2718euler
2012-02-23 20:58:07
n=32
n=32
iamnew
2012-02-23 20:58:07
32
32
copeland
2012-02-23 20:58:13
The next sequence I wanted to look at was the one that starts with N=32. What does that sequence look like?
The next sequence I wanted to look at was the one that starts with N=32. What does that sequence look like?
MathForFun
2012-02-23 20:59:46
81
81
pr0likethis
2012-02-23 20:59:46
3^4=81
3^4=81
lucylai
2012-02-23 20:59:46
32, 81, 64, 243, ...
32, 81, 64, 243, ...
.cpp
2012-02-23 20:59:46
32 => 3^4 => 4^3 = 2^6 => 3^5 => 4^4 = 2^8 ...
32 => 3^4 => 4^3 = 2^6 => 3^5 => 4^4 = 2^8 ...
PChang
2012-02-23 20:59:46
3^4,4^3,3^5,... soon
3^4,4^3,3^5,... soon
fractals
2012-02-23 20:59:46
2^5 3^4 4^3=2^6 3^5 4^4=2^8, ..., and it seems unbounded
2^5 3^4 4^3=2^6 3^5 4^4=2^8, ..., and it seems unbounded
Goos
2012-02-23 20:59:46
81, 64, 3^5, 2^8,... unbounded!
81, 64, 3^5, 2^8,... unbounded!
number.sense
2012-02-23 20:59:46
32, 81, 64, 243...
32, 81, 64, 243...
copeland
2012-02-23 20:59:55
It looks like these are unbounded. Therefore 32 is our first case of an unbounded sequence, hopefully. Maybe it'll shed some light to prove that.
It looks like these are unbounded. Therefore 32 is our first case of an unbounded sequence, hopefully. Maybe it'll shed some light to prove that.
2718euler
2012-02-23 21:00:46
4^(n-2)
4^(n-2)
.cpp
2012-02-23 21:00:46
4^(n-2)
4^(n-2)
lucylai
2012-02-23 21:00:46
4^(n-2)
4^(n-2)
nackster12
2012-02-23 21:00:46
2^(2n-4)
2^(2n-4)
PChang
2012-02-23 21:00:46
2^(n-2)sqrd
2^(n-2)sqrd
pgmath
2012-02-23 21:00:46
2^(2n-4)
2^(2n-4)
infinity1
2012-02-23 21:00:46
2^2n-4
2^2n-4
pr0likethis
2012-02-23 21:01:27
n>4
n>4
joshxiong
2012-02-23 21:01:27
n>4
n>4
infinity1
2012-02-23 21:01:27
n>4
n>4
nikeballa96
2012-02-23 21:01:27
n<2n-4 so n>4
n<2n-4 so n>4
pr0likethis
2012-02-23 21:01:27
so n can be anything from 5 to 8
so n can be anything from 5 to 8
number.sense
2012-02-23 21:01:27
when 2n>4
when 2n>4
.cpp
2012-02-23 21:01:27
2n-4 > n => n>4,
2n-4 > n => n>4,
Goos
2012-02-23 21:01:27
2n-4>n, so n>4
2n-4>n, so n>4
tan90
2012-02-23 21:01:27
n>=5
n>=5
pgmath
2012-02-23 21:01:27
n>4
n>4
copeland
2012-02-23 21:01:31
If n>4 then 2n-4 > n. Therefore this sequence is unbounded whenever we start with a power of 2 that is at least 2^5=32.
If n>4 then 2n-4 > n. Therefore this sequence is unbounded whenever we start with a power of 2 that is at least 2^5=32.
copeland
2012-02-23 21:01:34
Do we get any other unbounded sequences from knowing that 32 starts an unbounded sequence?
Do we get any other unbounded sequences from knowing that 32 starts an unbounded sequence?
pr0likethis
2012-02-23 21:02:33
any multiple of 32!
any multiple of 32!
ewcikewqikd
2012-02-23 21:02:33
all multiple of 32
all multiple of 32
PChang
2012-02-23 21:02:33
and multiples of those
and multiples of those
joshxiong
2012-02-23 21:02:33
All other numbers in that sequnce and their multiples.
All other numbers in that sequnce and their multiples.
canadian
2012-02-23 21:02:33
Any multiple of 32 also starts an unbounded sequence.
Any multiple of 32 also starts an unbounded sequence.
copeland
2012-02-23 21:02:36
It turns out that if a|b then f(a)|f(b).
It turns out that if a|b then f(a)|f(b).
copeland
2012-02-23 21:02:51
If all the exponents in a are at least as large as all the exponents in b then all the exponents in f(a) will be at least as large as the exponents in f(b).
If all the exponents in a are at least as large as all the exponents in b then all the exponents in f(a) will be at least as large as the exponents in f(b).
copeland
2012-02-23 21:02:54
In particular, all the multiples of 32 will start unbounded sequences. How many multiples of 32 are there between 1 and 400?
In particular, all the multiples of 32 will start unbounded sequences. How many multiples of 32 are there between 1 and 400?
super_pi314
2012-02-23 21:03:26
12
12
Yongyi781
2012-02-23 21:03:26
12
12
aerrowfinn72
2012-02-23 21:03:28
12
12
Iggy Iguana
2012-02-23 21:03:28
12
12
coldsummer
2012-02-23 21:03:28
12
12
pattycakechichi
2012-02-23 21:03:28
12
12
soccerfan
2012-02-23 21:03:28
12
12
matemyday1234567890
2012-02-23 21:03:28
12
12
RotomPlasma
2012-02-23 21:03:28
12
12
nackster12
2012-02-23 21:03:28
12
12
copeland
2012-02-23 21:03:35
What else does the 32 sequence give us?
What else does the 32 sequence give us?
copeland
2012-02-23 21:03:48
Beyond multiples of 32 that is.
Beyond multiples of 32 that is.
christerson
2012-02-23 21:04:30
81
81
Seedleaf
2012-02-23 21:04:30
all multiples of 3^4
all multiples of 3^4
ewcikewqikd
2012-02-23 21:04:30
3^4 = 81
3^4 = 81
.cpp
2012-02-23 21:04:30
3^4 and any multiple of it.
3^4 and any multiple of it.
Iggy Iguana
2012-02-23 21:04:30
multiples of 81
multiples of 81
aopsvd
2012-02-23 21:04:30
Multiples of 3^4.
Multiples of 3^4.
munygrubber
2012-02-23 21:04:30
81 works too
81 works too
copeland
2012-02-23 21:04:33
Since 3^4=81 is in this sequence, 81 and all of its multiples start unbounded sequences.
Since 3^4=81 is in this sequence, 81 and all of its multiples start unbounded sequences.
copeland
2012-02-23 21:04:37
There are 4 multiples of 81 below 400: 81, 162, 243, and 324.
There are 4 multiples of 81 below 400: 81, 162, 243, and 324.
copeland
2012-02-23 21:04:40
Therefore we're up to 16 good values for N.
Therefore we're up to 16 good values for N.
copeland
2012-02-23 21:04:43
Powers of 2 and powers of 3 were nice. Let's try powers of 5.
Powers of 2 and powers of 3 were nice. Let's try powers of 5.
copeland
2012-02-23 21:04:47
We know that the N=5 sequence goes 5, 1, 1, 1,. . . .
We know that the N=5 sequence goes 5, 1, 1, 1,. . . .
copeland
2012-02-23 21:04:50
What about the N=25 sequence?
What about the N=25 sequence?
Iggy Iguana
2012-02-23 21:05:41
25, 6, 1
25, 6, 1
number.sense
2012-02-23 21:05:41
25, 6, 1,1,1,1
25, 6, 1,1,1,1
pgmath
2012-02-23 21:05:41
6, 1, 1, ...
6, 1, 1, ...
va2010
2012-02-23 21:05:41
25,6,loses
25,6,loses
Seedleaf
2012-02-23 21:05:41
25, 6, 1, 1...
25, 6, 1, 1...
mcdonalds106_7
2012-02-23 21:05:41
25,6,1,1,1,1,...
25,6,1,1,1,1,...
KeepingItReal
2012-02-23 21:05:41
5^2, 6,1,1,1...
5^2, 6,1,1,1...
Goos
2012-02-23 21:05:41
5^2, 6^1, 1,1,1,1
5^2, 6^1, 1,1,1,1
nackster12
2012-02-23 21:05:41
25,6,1
25,6,1
fractals
2012-02-23 21:05:41
5^2 6^1=2*3 1, so none
5^2 6^1=2*3 1, so none
copeland
2012-02-23 21:05:48
That's no good. What about N=125?
That's no good. What about N=125?
Iggy Iguana
2012-02-23 21:06:51
125, 36, 12, 3, 1
125, 36, 12, 3, 1
va2010
2012-02-23 21:06:51
125, 36, 12, loses
125, 36, 12, loses
mcdonalds106_7
2012-02-23 21:06:51
125,36,12,3,1,1,1,1,...
125,36,12,3,1,1,1,1,...
number.sense
2012-02-23 21:06:51
125, 36, 12, ... bounded
125, 36, 12, ... bounded
pgmath
2012-02-23 21:06:51
36, 12, 3, 1, 1, ...
36, 12, 3, 1, 1, ...
nackster12
2012-02-23 21:06:51
125,36,3,1
125,36,3,1
super_pi314
2012-02-23 21:06:51
125,36,12,3,1,1,1...
125,36,12,3,1,1,1...
soccerfan
2012-02-23 21:06:53
125, 36, 12, 3, 1...
125, 36, 12, 3, 1...
copeland
2012-02-23 21:07:02
OK, so powers of 5 are not helpful. What about powers of 7? We know 7 is no good. What is the sequence for N=49?
OK, so powers of 5 are not helpful. What about powers of 7? We know 7 is no good. What is the sequence for N=49?
Seedleaf
2012-02-23 21:07:57
49, 8, 9, 4, 1, ...
49, 8, 9, 4, 1, ...
MathForFun
2012-02-23 21:07:57
8, 9, 4, 3
8, 9, 4, 3
Iggy Iguana
2012-02-23 21:07:57
49, 8, 9, 4, 3, 1
49, 8, 9, 4, 3, 1
bookie331
2012-02-23 21:07:57
49, 8, 9, 4, 3, 1, 1
49, 8, 9, 4, 3, 1, 1
copeland
2012-02-23 21:08:08
However what is promising about 7?
However what is promising about 7?
Seedleaf
2012-02-23 21:08:39
it gives powers of 2. try 343?
it gives powers of 2. try 343?
Yongyi781
2012-02-23 21:08:39
7+1 = 8 which is 2^3
7+1 = 8 which is 2^3
.cpp
2012-02-23 21:08:39
Powers of 2!
Powers of 2!
Goos
2012-02-23 21:08:39
it goes to 8 which is a power of 2! 343 works.
it goes to 8 which is a power of 2! 343 works.
number.sense
2012-02-23 21:08:39
it gives us 8, which is 2^3 :D which gives us large powers of 2
it gives us 8, which is 2^3 :D which gives us large powers of 2
nackster12
2012-02-23 21:08:39
you get powers of 2
you get powers of 2
copeland
2012-02-23 21:08:46
7 is promising because it gives a great big power of 2.
7 is promising because it gives a great big power of 2.
copeland
2012-02-23 21:09:02
There is only one multiple of this number that is less than 400.
There is only one multiple of this number that is less than 400.
copeland
2012-02-23 21:09:06
Our count of Ns that work is now up to 17. That's up there on the list, but it might be a distractor. After all we've only thought about prime powers so far. Now we should think about products of prime powers.
Our count of Ns that work is now up to 17. That's up there on the list, but it might be a distractor. After all we've only thought about prime powers so far. Now we should think about products of prime powers.
number.sense
2012-02-23 21:09:41
1
1
pgmath
2012-02-23 21:09:41
1
1
Yongyi781
2012-02-23 21:09:41
1
1
fractals
2012-02-23 21:09:41
1
1
number.sense
2012-02-23 21:09:41
1
1
alex31415
2012-02-23 21:09:41
1?
1?
lucylai
2012-02-23 21:09:41
1
1
copeland
2012-02-23 21:09:52
So what should we really care about?
So what should we really care about?
MathForFun
2012-02-23 21:10:30
The exponents
The exponents
Seedleaf
2012-02-23 21:10:30
the bigger powers
the bigger powers
pr0likethis
2012-02-23 21:10:30
squares of primes
squares of primes
pgmath
2012-02-23 21:10:30
Products of squares
Products of squares
aopsvd
2012-02-23 21:10:30
multiples of squares by factors of those squares.
multiples of squares by factors of those squares.
ksun48
2012-02-23 21:10:30
at least squares
at least squares
va2010
2012-02-23 21:10:30
products of squares
products of squares
.cpp
2012-02-23 21:10:30
We need squares!
We need squares!
timoteomo3
2012-02-23 21:10:30
the largest exponent!
the largest exponent!
copeland
2012-02-23 21:10:40
The factors that are going to matter in the end are the primes with exponent greater than 1. If N is not divisible by 3 then f_1(3N)=f_1(N), so the sequence for 3N is the same as the sequence for N. Therefore we should narrow our search to numbers that have exponents that are at least 2.
The factors that are going to matter in the end are the primes with exponent greater than 1. If N is not divisible by 3 then f_1(3N)=f_1(N), so the sequence for 3N is the same as the sequence for N. Therefore we should narrow our search to numbers that have exponents that are at least 2.
va2010
2012-02-23 21:11:49
2
2
.cpp
2012-02-23 21:11:49
Can't be too big.
Can't be too big.
zinko1991
2012-02-23 21:11:49
2?
2?
Yongyi781
2012-02-23 21:11:49
2
2
copeland
2012-02-23 21:11:59
Let's start with 2 and 3. We know that any multiple of 2^5 or 3^4 will give an unbounded sequence. Therefore we ought to think about the number 2^43^3. If this number's sequence is bounded then any factor of this number will have a bounded sequence as well.
Let's start with 2 and 3. We know that any multiple of 2^5 or 3^4 will give an unbounded sequence. Therefore we ought to think about the number 2^43^3. If this number's sequence is bounded then any factor of this number will have a bounded sequence as well.
Iggy Iguana
2012-02-23 21:13:10
2^4*3^3
2^4*3^3
PChang
2012-02-23 21:13:10
same thing!!!
same thing!!!
MathForFun
2012-02-23 21:13:10
432
432
KeepingItReal
2012-02-23 21:13:10
3^3 * 4^2
3^3 * 4^2
Seedleaf
2012-02-23 21:13:10
2^4*3^3
2^4*3^3
Yongyi781
2012-02-23 21:13:10
3^3 4^2 = 2^4 3^3
3^3 4^2 = 2^4 3^3
bookie331
2012-02-23 21:13:10
432
432
mz94
2012-02-23 21:13:10
itself; it cycles so its bounded
itself; it cycles so its bounded
aopsvd
2012-02-23 21:13:10
3^3 2^4
3^3 2^4
Goos
2012-02-23 21:13:10
3^3*2^4
3^3*2^4
copeland
2012-02-23 21:13:27
Some people are asking why we tested this one.
Some people are asking why we tested this one.
copeland
2012-02-23 21:13:51
Any number that's a factor of this one cannot possibly generate an unbounded sequence.
Any number that's a factor of this one cannot possibly generate an unbounded sequence.
copeland
2012-02-23 21:14:03
We now that if a number generates an unbounded sequence then any multiple of it will as well.
We now that if a number generates an unbounded sequence then any multiple of it will as well.
copeland
2012-02-23 21:14:23
So by testing this guy we've simultaneously tested all other numbers of the form 3^a2^b.
So by testing this guy we've simultaneously tested all other numbers of the form 3^a2^b.
copeland
2012-02-23 21:14:34
OK, fine. The problem here, though, was that applying f_1 takes powers of 3 to powers of 2 and powers of 2 to powers of 3. They never had a chance to combine and make a bigger number.
OK, fine. The problem here, though, was that applying f_1 takes powers of 3 to powers of 2 and powers of 2 to powers of 3. They never had a chance to combine and make a bigger number.
copeland
2012-02-23 21:14:43
Let's try 2 and 5 next. Since 2^25^2=100, we can't use any more fives in our number. Therefore we should think about numbers of the form 2^e5^2.
Let's try 2 and 5 next. Since 2^25^2=100, we can't use any more fives in our number. Therefore we should think about numbers of the form 2^e5^2.
number.sense
2012-02-23 21:15:33
3^e * 2
3^e * 2
ksun48
2012-02-23 21:15:33
3^e*2
3^e*2
pgmath
2012-02-23 21:15:33
3^e * 2
3^e * 2
.cpp
2012-02-23 21:15:33
3^(e)*2
3^(e)*2
pr0likethis
2012-02-23 21:15:33
6*3^e-1 = 2*3^e
6*3^e-1 = 2*3^e
nackster12
2012-02-23 21:15:33
2*3^e
2*3^e
copeland
2012-02-23 21:15:48
We can apply f again to get
We can apply f again to get
copeland
2012-02-23 21:15:52
For what e is sequence unbounded?
For what e is sequence unbounded?
pr0likethis
2012-02-23 21:16:32
so e=4 makes N=400, which works :D
so e=4 makes N=400, which works :D
ewcikewqikd
2012-02-23 21:16:32
4
4
alex31415
2012-02-23 21:16:32
4
4
MathForFun
2012-02-23 21:16:32
e>3/2
e>3/2
ksun48
2012-02-23 21:16:34
e >= 4
e >= 4
pgmath
2012-02-23 21:16:34
e = 4
e = 4
fractals
2012-02-23 21:16:34
e >= 4
e >= 4
ksun48
2012-02-23 21:16:34
so 400
so 400
copeland
2012-02-23 21:16:47
Check it out! 400=2^4*5^2. It's the end of the range. Therefore we've found another valid N. That bumps the count up to 18.
Check it out! 400=2^4*5^2. It's the end of the range. Therefore we've found another valid N. That bumps the count up to 18.
copeland
2012-02-23 21:17:14
That's also annoying because there might be one more solution lurking around. . . .
That's also annoying because there might be one more solution lurking around. . . .
copeland
2012-02-23 21:17:58
So what is the final answer?
So what is the final answer?
ahaanomegas
2012-02-23 21:18:45
(D)
(D)
RotomPlasma
2012-02-23 21:18:45
so (D)!
so (D)!
Iggy Iguana
2012-02-23 21:18:45
D)18
D)18
MathForFun
2012-02-23 21:18:45
D
D
coldsummer
2012-02-23 21:18:45
D
D
pgmath
2012-02-23 21:18:45
(D) 18
(D) 18
Seedleaf
2012-02-23 21:18:45
D: 18
D: 18
ksun48
2012-02-23 21:18:45
D18
D18
number.sense
2012-02-23 21:18:45
D
D
MathForFun
2012-02-23 21:18:45
D?
D?
pr0likethis
2012-02-23 21:18:45
18 (D). 400 was very difficult to find though....
18 (D). 400 was very difficult to find though....
va2010
2012-02-23 21:18:45
18 /D
18 /D
RunpengFAILS
2012-02-23 21:18:45
Eighteen
Eighteen
pattycakechichi
2012-02-23 21:18:45
18
18
alex31415
2012-02-23 21:18:45
18?
18?
pattycakechichi
2012-02-23 21:18:45
D
D
fractals
2012-02-23 21:18:45
18, D
18, D
copeland
2012-02-23 21:18:47
We've found 18 total values of N that satisfy the problem so the answer is (D).
We've found 18 total values of N that satisfy the problem so the answer is (D).
copeland
2012-02-23 21:19:07
Good work!
Good work!
copeland
2012-02-23 21:19:09
We are awesome.
We are awesome.
Iggy Iguana
2012-02-23 21:19:13
that took a long time
that took a long time
number.sense
2012-02-23 21:19:13
... too long...
... too long...
copeland
2012-02-23 21:19:21
Yeah, but that problem was like the 25 on this test.
Yeah, but that problem was like the 25 on this test.
copeland
2012-02-23 21:19:28
And we'll call the 25 a 23. And we'll call 23 a 30.
And we'll call the 25 a 23. And we'll call 23 a 30.
matemyday1234567890
2012-02-23 21:19:39
how do you solve that quicker
how do you solve that quicker
copeland
2012-02-23 21:19:43
You can't, really.
You can't, really.
copeland
2012-02-23 21:19:47
You can guess better than we did.
You can guess better than we did.
copeland
2012-02-23 21:19:58
Thinking about parity helps.
Thinking about parity helps.
copeland
2012-02-23 21:20:43
Realizing that 7 and 3 are like anti-twos and that 5 is both a 2 and an anti-two put together.
Realizing that 7 and 3 are like anti-twos and that 5 is both a 2 and an anti-two put together.
copeland
2012-02-23 21:20:46
That kind of thing.
That kind of thing.
copeland
2012-02-23 21:20:50
But it only gets you so far.
But it only gets you so far.
copeland
2012-02-23 21:21:08
And the problem came with a nice picture of S so that you don't actually have to figure out what all that notation means:
And the problem came with a nice picture of S so that you don't actually have to figure out what all that notation means:
copeland
2012-02-23 21:21:22
First off, what is the value of f(t) for this right triangle?
First off, what is the value of f(t) for this right triangle?
fractals
2012-02-23 21:21:58
3/4
3/4
Yongyi781
2012-02-23 21:21:58
3/4
3/4
alex31415
2012-02-23 21:21:58
3/4
3/4
KeepingItReal
2012-02-23 21:21:58
3/4
3/4
RotomPlasma
2012-02-23 21:21:58
3/4
3/4
copeland
2012-02-23 21:22:08
And the value of the tangent at angle B is 3/4.
And the value of the tangent at angle B is 3/4.
copeland
2012-02-23 21:22:10
What is the value of f for this triangle?
What is the value of f for this triangle?
ksun48
2012-02-23 21:22:35
4/3
4/3
fractals
2012-02-23 21:22:35
4/3
4/3
KeepingItReal
2012-02-23 21:22:35
4/3
4/3
Iggy Iguana
2012-02-23 21:22:35
4/3
4/3
lucylai
2012-02-23 21:22:35
3/4
3/4
joshxiong
2012-02-23 21:22:35
4/3
4/3
noedne
2012-02-23 21:22:35
4/3
4/3
nackster12
2012-02-23 21:22:35
4/3
4/3
pgmath
2012-02-23 21:22:35
4/3
4/3
copeland
2012-02-23 21:22:41
Again we label:
Again we label:
copeland
2012-02-23 21:22:47
And the tangent of B is 4/3.
And the tangent of B is 4/3.
copeland
2012-02-23 21:22:50
These are reflections of one another and the values of f are reciprocals.
These are reflections of one another and the values of f are reciprocals.
copeland
2012-02-23 21:22:55
That's nice. There's some symmetry in the problem, but there's not really any symmetry in the question because we're missing the point (0,0).
That's nice. There's some symmetry in the problem, but there's not really any symmetry in the question because we're missing the point (0,0).
copeland
2012-02-23 21:23:00
Does this suggest any approach?
Does this suggest any approach?
number.sense
2012-02-23 21:23:55
add the point 0,0
add the point 0,0
Yongyi781
2012-02-23 21:23:55
Pretend (0,0) is there and then account for the triangles that contain (0,0)
Pretend (0,0) is there and then account for the triangles that contain (0,0)
canadian
2012-02-23 21:23:55
exclusion
exclusion
fractals
2012-02-23 21:23:55
Pretend (0,0) is in the set and the divide for the overcounting
Pretend (0,0) is in the set and the divide for the overcounting
Goos
2012-02-23 21:23:55
look at triangles whose counterparts (reflections) go through (0,0).
look at triangles whose counterparts (reflections) go through (0,0).
alex31415
2012-02-23 21:23:55
try including (0,0) and subtract the cases with (0,0)
try including (0,0) and subtract the cases with (0,0)
copeland
2012-02-23 21:23:59
Let's solve a simpler problem again. Throw the point (0,0) into the mix.
Let's solve a simpler problem again. Throw the point (0,0) into the mix.
copeland
2012-02-23 21:24:13
How can we use the symmetry that we've found?
How can we use the symmetry that we've found?
KeepingItReal
2012-02-23 21:24:52
each triangle has a conjugate, so tangents multiply to 1
each triangle has a conjugate, so tangents multiply to 1
fractals
2012-02-23 21:24:55
Pair up triangles by a reflection over x = 2
Pair up triangles by a reflection over x = 2
Yongyi781
2012-02-23 21:24:55
Can we pair up every triangle with its "conjugate"?
Can we pair up every triangle with its "conjugate"?
pgmath
2012-02-23 21:24:55
Every triangle has a reflection that it cancels out with.
Every triangle has a reflection that it cancels out with.
ewcikewqikd
2012-02-23 21:24:55
1 because any right triangle reflect over the middle vertical line cancels itself
1 because any right triangle reflect over the middle vertical line cancels itself
number.sense
2012-02-23 21:24:55
every triangle has its reflection over the line through the middle of the grid
every triangle has its reflection over the line through the middle of the grid
newchie123
2012-02-23 21:24:55
since they cancel repricocals
since they cancel repricocals
copeland
2012-02-23 21:25:00
If we reflect our picture across the vertical midline, every right triangle is paired with another right triangle with a value of f that is inverse to the value of the original triangle.
If we reflect our picture across the vertical midline, every right triangle is paired with another right triangle with a value of f that is inverse to the value of the original triangle.
copeland
2012-02-23 21:25:05
Except not quite. Which triangles have no partners?
Except not quite. Which triangles have no partners?
nackster12
2012-02-23 21:25:59
the ones that are symmetrical about the center line
the ones that are symmetrical about the center line
number.sense
2012-02-23 21:25:59
isosceles right triangles with right angle perfectly centered around our axis of symmetry
isosceles right triangles with right angle perfectly centered around our axis of symmetry
Yongyi781
2012-02-23 21:25:59
The ones with the midline as an axis of symmetry
The ones with the midline as an axis of symmetry
Goos
2012-02-23 21:25:59
triangles symmetrical about the midline. This means they have a right angle along the midline and two points on either side.
triangles symmetrical about the midline. This means they have a right angle along the midline and two points on either side.
pgmath
2012-02-23 21:26:02
The isosceles ones with a vertex on the middle column
The isosceles ones with a vertex on the middle column
copeland
2012-02-23 21:26:06
There are some triangles that are fixed by this reflection:
There are some triangles that are fixed by this reflection:
copeland
2012-02-23 21:26:12
However what do we know about f applied to these triangles?
However what do we know about f applied to these triangles?
Yongyi781
2012-02-23 21:26:33
But isosceles -> tan = 1 so that doesn't matter
But isosceles -> tan = 1 so that doesn't matter
ewcikewqikd
2012-02-23 21:26:33
equals 1
equals 1
c0mbusti0n1295
2012-02-23 21:26:33
always 1
always 1
fractals
2012-02-23 21:26:33
But those are 45-45-90, which have tangent 1, so they don't matter
But those are 45-45-90, which have tangent 1, so they don't matter
noedne
2012-02-23 21:26:33
it equals 1
it equals 1
joshxiong
2012-02-23 21:26:33
It is 1.
It is 1.
DF2222
2012-02-23 21:26:33
it's 1
it's 1
Iggy Iguana
2012-02-23 21:26:33
its 1!
its 1!
number.sense
2012-02-23 21:26:33
it already is 1
it already is 1
bookie331
2012-02-23 21:26:33
1
1
pr0likethis
2012-02-23 21:26:33
f is 1, so it doesnt need canceling
f is 1, so it doesnt need canceling
distortedwalrus
2012-02-23 21:26:33
it's 1
it's 1
copeland
2012-02-23 21:26:36
Any triangle fixed by this reflection is isosceles and the value of f on an isosceles triangle must be 1. (Alternatively, since these triangles are fixed by a reflection but f(t) is the reciprocal of the value of f on the reflection of t, we have to have f(t)=1 for these triangles.)
Any triangle fixed by this reflection is isosceles and the value of f on an isosceles triangle must be 1. (Alternatively, since these triangles are fixed by a reflection but f(t) is the reciprocal of the value of f on the reflection of t, we have to have f(t)=1 for these triangles.)
copeland
2012-02-23 21:26:40
Therefore what is the value of the big product we seek on this new S where we include the origin?
Therefore what is the value of the big product we seek on this new S where we include the origin?
lucylai
2012-02-23 21:26:57
it's just 1
it's just 1
Seedleaf
2012-02-23 21:26:57
1
1
number.sense
2012-02-23 21:26:57
1
1
KeepingItReal
2012-02-23 21:26:57
1
1
noedne
2012-02-23 21:26:57
1
1
nackster12
2012-02-23 21:26:57
1
1
RunpengFAILS
2012-02-23 21:26:57
One.
One.
mcdonalds106_7
2012-02-23 21:26:57
1
1
copeland
2012-02-23 21:27:03
The value is 1. The symmetric triangles don't contribute to the product and the other triangles pair off into reciprocal factors. Everything cancels leaving only the value 1.
The value is 1. The symmetric triangles don't contribute to the product and the other triangles pair off into reciprocal factors. Everything cancels leaving only the value 1.
copeland
2012-02-23 21:27:06
How can we use this to simplify the original problem?
How can we use this to simplify the original problem?
nackster12
2012-02-23 21:27:34
find all the triangles that have a vertex at the origin
find all the triangles that have a vertex at the origin
narto928
2012-02-23 21:27:34
just account for the ones that contain (0,0)
just account for the ones that contain (0,0)
number.sense
2012-02-23 21:27:34
how many triangles now use the origin is the only question left to answer
how many triangles now use the origin is the only question left to answer
pr0likethis
2012-02-23 21:27:34
our answer is the inverse of all f(t) with t including the origin
our answer is the inverse of all f(t) with t including the origin
varunrocks
2012-02-23 21:27:34
look at all the triangles that pass through the origin and just take the reciprocal
look at all the triangles that pass through the origin and just take the reciprocal
copeland
2012-02-23 21:27:38
Since we know the product for all of the triangles is 1, and we want the product for all triangles that don't use the origin, we can compute the product for the triangles that do use the origin and take its reciprocal.
Since we know the product for all of the triangles is 1, and we want the product for all triangles that don't use the origin, we can compute the product for the triangles that do use the origin and take its reciprocal.
copeland
2012-02-23 21:27:43
(At this point when solving, I looked to see if any of the choices were reciprocals of one another to see if I had to pay attention to that. PHEW! None of them are.)
(At this point when solving, I looked to see if any of the choices were reciprocals of one another to see if I had to pay attention to that. PHEW! None of them are.)
copeland
2012-02-23 21:27:53
Now we've reduced the problem to finding the value of the product for all triangles that use the origin (0,0). There are still a TON of them. Is there another reflection symmetry that might help?
Now we've reduced the problem to finding the value of the product for all triangles that use the origin (0,0). There are still a TON of them. Is there another reflection symmetry that might help?
copeland
2012-02-23 21:28:30
Notice here that we want a symmetry that permutes the set of triangles containing the origin as a vertex. Therefore we want a reflection that fixes the origin and mostly preserves the set of triangles. We can't find a symmetry that does both, but preserving the origin is a LOT more important.
Notice here that we want a symmetry that permutes the set of triangles containing the origin as a vertex. Therefore we want a reflection that fixes the origin and mostly preserves the set of triangles. We can't find a symmetry that does both, but preserving the origin is a LOT more important.
dinoboy
2012-02-23 21:28:54
x=y
x=y
KeepingItReal
2012-02-23 21:28:54
reflect the triangles about y=x?
reflect the triangles about y=x?
dinoboy
2012-02-23 21:28:54
*x=y, except when some vertex on the top is used
*x=y, except when some vertex on the top is used
canadian
2012-02-23 21:28:54
Along the diagonal
Along the diagonal
pr0likethis
2012-02-23 21:28:54
reflecting over the "line" y=x creates canceling triangles as well
reflecting over the "line" y=x creates canceling triangles as well
Goos
2012-02-23 21:28:54
reflection about the diagonal (y=x, if it were the first quadrant.)
reflection about the diagonal (y=x, if it were the first quadrant.)
copeland
2012-02-23 21:28:58
Let's consider the reflection through this line:
Let's consider the reflection through this line:
copeland
2012-02-23 21:29:05
What triangles are paired by this reflection?
What triangles are paired by this reflection?
fractals
2012-02-23 21:29:55
All but those with a vertex in the top row
All but those with a vertex in the top row
mcdonalds106_7
2012-02-23 21:29:55
ones that don't have a vertex on y=5
ones that don't have a vertex on y=5
Goos
2012-02-23 21:29:55
Anything not including the top five points.
Anything not including the top five points.
copeland
2012-02-23 21:30:03
Any triangle that doesn't have a vertex along the top row will have a partner triangle under this reflection.
Any triangle that doesn't have a vertex along the top row will have a partner triangle under this reflection.
copeland
2012-02-23 21:30:12
So?
So?
narto928
2012-02-23 21:31:02
we find the ones that use the top row
we find the ones that use the top row
nackster12
2012-02-23 21:31:02
all triangles with a vertex at the origin and in the top row
all triangles with a vertex at the origin and in the top row
number.sense
2012-02-23 21:31:02
we merely have to divide out those that use the top row?
we merely have to divide out those that use the top row?
KeepingItReal
2012-02-23 21:31:02
find ones with vertex on top row
find ones with vertex on top row
lucylai
2012-02-23 21:31:02
their tangents multiply to 1
their tangents multiply to 1
fractals
2012-02-23 21:31:02
Now we count those with one vertex on the origin and at least one in the top row.
Now we count those with one vertex on the origin and at least one in the top row.
Goos
2012-02-23 21:31:02
We know that 1 point is (0,0) and the other is on the top of the grid. It's pretty simple from there.
We know that 1 point is (0,0) and the other is on the top of the grid. It's pretty simple from there.
copeland
2012-02-23 21:31:12
Those triangles also cancel in our product.
Those triangles also cancel in our product.
copeland
2012-02-23 21:31:16
That leaves only triangles that use the origin and use at least one of the topmost vertices. Now we're in business.
That leaves only triangles that use the origin and use at least one of the topmost vertices. Now we're in business.
copeland
2012-02-23 21:31:19
We seem to have 4 types of such triangles. We have triangles with legs along the x- and y- axis:
We seem to have 4 types of such triangles. We have triangles with legs along the x- and y- axis:
copeland
2012-02-23 21:31:31
We also have triangles with legs along the left side and along the top:
We also have triangles with legs along the left side and along the top:
copeland
2012-02-23 21:31:40
We also have triangles with legs parallel to the axes but right angle elsewhere in the grid:
We also have triangles with legs parallel to the axes but right angle elsewhere in the grid:
copeland
2012-02-23 21:31:50
Finally we have triangles that are just kinda wonky:
Finally we have triangles that are just kinda wonky:
copeland
2012-02-23 21:31:58
(Those purple guys make me nervous.)
(Those purple guys make me nervous.)
copeland
2012-02-23 21:32:04
Is there any more symmetry that we can take advantage of?
Is there any more symmetry that we can take advantage of?
KeepingItReal
2012-02-23 21:33:04
for the first three cases, yes. those *should* cancel i think?
for the first three cases, yes. those *should* cancel i think?
pr0likethis
2012-02-23 21:33:04
each case that isnt purple has inverses
each case that isnt purple has inverses
fractals
2012-02-23 21:33:04
Reds and blues are reflections of each other and therefore cancel out (to 1).
Reds and blues are reflections of each other and therefore cancel out (to 1).
copeland
2012-02-23 21:33:06
Do those all just cancel and go away?
Do those all just cancel and go away?
shengo8
2012-02-23 21:33:28
no
no
zinko1991
2012-02-23 21:33:28
no
no
copeland
2012-02-23 21:33:44
But some of them cancel, right?
But some of them cancel, right?
sammyMaX
2012-02-23 21:34:02
No, two of them cancel out, leaving one behind along with the purple ones
No, two of them cancel out, leaving one behind along with the purple ones
pr0likethis
2012-02-23 21:34:02
oh, there are three of them. one pair cancels, but one remains
oh, there are three of them. one pair cancels, but one remains
copeland
2012-02-23 21:34:06
Actually any 2 of the first three sets can be paired off by some reflection. Let's get rid of the first two (because slopes in the third case look easy to compute). The first two are interchanged by the reflection through the horizontal midline so they cancel.
Actually any 2 of the first three sets can be paired off by some reflection. Let's get rid of the first two (because slopes in the third case look easy to compute). The first two are interchanged by the reflection through the horizontal midline so they cancel.
copeland
2012-02-23 21:34:12
Now we are left computing the product for triangles of the form
Now we are left computing the product for triangles of the form
copeland
2012-02-23 21:34:21
and
and
copeland
2012-02-23 21:34:28
What is f applied to the blue triangle?
What is f applied to the blue triangle?
fractals
2012-02-23 21:35:04
3/5
3/5
KeepingItReal
2012-02-23 21:35:04
3/5
3/5
pr0likethis
2012-02-23 21:35:04
n/5 where n is 1, 2, 3, or 4
n/5 where n is 1, 2, 3, or 4
MathForFun
2012-02-23 21:35:04
3/5
3/5
Iggy Iguana
2012-02-23 21:35:04
3/5
3/5
dsj9999
2012-02-23 21:35:10
3/5
3/5
copeland
2012-02-23 21:35:11
We label the vertices counterclockwise from the right angle and then we compute the ratio of the opposite side to the adjacent side. The adjacent side is 5 and the opposite side is 3. The blue triangle has f(t)=3/5.
We label the vertices counterclockwise from the right angle and then we compute the ratio of the opposite side to the adjacent side. The adjacent side is 5 and the opposite side is 3. The blue triangle has f(t)=3/5.
copeland
2012-02-23 21:35:24
Super. We've discovered one of the distractors! However we need to deal with the wonky triangles, too. The wonky triangles have right angle somewhere in the middle of the diagram. What relationship is there between the legs of a wonky triangle that will let us locate the point with the right angle?
Super. We've discovered one of the distractors! However we need to deal with the wonky triangles, too. The wonky triangles have right angle somewhere in the middle of the diagram. What relationship is there between the legs of a wonky triangle that will let us locate the point with the right angle?
sammyMaX
2012-02-23 21:36:18
The slopes of the legs are negative reciprocals
The slopes of the legs are negative reciprocals
nackster12
2012-02-23 21:36:18
opposite reciprocoal slopes
opposite reciprocoal slopes
c0mbusti0n1295
2012-02-23 21:36:18
slopes are negative reciprocals?
slopes are negative reciprocals?
copeland
2012-02-23 21:36:26
The slopes of the legs are negative reciprocals!
The slopes of the legs are negative reciprocals!
copeland
2012-02-23 21:36:30
How many wonky triangles are there with slopes +1 and -1?
How many wonky triangles are there with slopes +1 and -1?
mcdonalds106_7
2012-02-23 21:37:26
2
2
fractals
2012-02-23 21:37:26
2
2
Iggy Iguana
2012-02-23 21:37:26
2.
2.
KeepingItReal
2012-02-23 21:37:26
2
2
billybob42
2012-02-23 21:37:26
2
2
canadian
2012-02-23 21:37:26
2
2
copeland
2012-02-23 21:37:29
The pink triangle above is one of them. Here's the other:
The pink triangle above is one of them. Here's the other:
copeland
2012-02-23 21:37:35
What is t for this second triangle?
What is t for this second triangle?
copeland
2012-02-23 21:37:40
f
f
copeland
2012-02-23 21:37:46
What is f for this second triangle?
What is f for this second triangle?
sammyMaX
2012-02-23 21:37:58
4
4
Iggy Iguana
2012-02-23 21:37:58
4.
4.
KeepingItReal
2012-02-23 21:37:58
4
4
RunpengFAILS
2012-02-23 21:37:58
Four.
Four.
joshxiong
2012-02-23 21:37:58
4
4
number.sense
2012-02-23 21:37:58
4
4
alex31415
2012-02-23 21:37:58
4?
4?
copeland
2012-02-23 21:38:06
What is f for this triangle:
What is f for this triangle:
alex31415
2012-02-23 21:38:44
3/2
3/2
joshxiong
2012-02-23 21:38:44
3/2
3/2
sammyMaX
2012-02-23 21:38:44
3/2
3/2
fractals
2012-02-23 21:38:44
3/2
3/2
bookfan
2012-02-23 21:38:44
3/2
3/2
dsj9999
2012-02-23 21:38:44
3/2
3/2
ABCDE
2012-02-23 21:38:44
3/2
3/2
RunpengFAILS
2012-02-23 21:38:44
Three halves
Three halves
Yongyi781
2012-02-23 21:38:44
3/2
3/2
Seedleaf
2012-02-23 21:38:44
3/2
3/2
billybob42
2012-02-23 21:38:44
3/2
3/2
aopsvd
2012-02-23 21:38:44
3/2
3/2
MathForFun
2012-02-23 21:38:44
3/2
3/2
distortedwalrus
2012-02-23 21:38:44
3/2
3/2
KeepingItReal
2012-02-23 21:38:44
3/2
3/2
copeland
2012-02-23 21:38:52
What are all the wonky triangles that we can make with slope 1/2 and 22 or with slope 2 and -1/2?
What are all the wonky triangles that we can make with slope 1/2 and 22 or with slope 2 and -1/2?
copeland
2012-02-23 21:39:11
What are all the wonky triangles that we can make with slope 1/2 and 2 or with slope 2 and -1/2?
What are all the wonky triangles that we can make with slope 1/2 and 2 or with slope 2 and -1/2?
copeland
2012-02-23 21:39:53
aargh!
aargh!
copeland
2012-02-23 21:39:59
What are all the wonky triangles that we can make with slope 1/2 and -2 or with slope 2 and -1/2?
What are all the wonky triangles that we can make with slope 1/2 and -2 or with slope 2 and -1/2?
KeepingItReal
2012-02-23 21:40:04
2
2
Iggy Iguana
2012-02-23 21:40:04
just the sqrt5 by 2sqrt5 one
just the sqrt5 by 2sqrt5 one
RotomPlasma
2012-02-23 21:40:04
2
2
KeepingItReal
2012-02-23 21:40:04
1 for each pair of slopes.
1 for each pair of slopes.
nackster12
2012-02-23 21:40:04
there are 2 of them
there are 2 of them
copeland
2012-02-23 21:40:08
Here are the only two:
Here are the only two:
copeland
2012-02-23 21:40:18
Do we care?
Do we care?
Iggy Iguana
2012-02-23 21:40:41
but they multiply to 1
but they multiply to 1
Seedleaf
2012-02-23 21:40:41
no, they cancel
no, they cancel
vcez
2012-02-23 21:40:41
no they cancel
no they cancel
Yongyi781
2012-02-23 21:40:41
No! They're conjugates
No! They're conjugates
fractals
2012-02-23 21:40:41
No, they multiply to 1
No, they multiply to 1
RotomPlasma
2012-02-23 21:40:41
No, symmetrical
No, symmetrical
c0mbusti0n1295
2012-02-23 21:40:41
they are reflections!
they are reflections!
copeland
2012-02-23 21:40:45
No! These two will cancel in the product.
No! These two will cancel in the product.
copeland
2012-02-23 21:40:49
How about slopes of n and 1/n for n>2?
How about slopes of n and 1/n for n>2?
KeepingItReal
2012-02-23 21:42:10
oh the one with slope 1/4 and -4
oh the one with slope 1/4 and -4
nackster12
2012-02-23 21:42:10
theres one with slopes 1/4 and -4
theres one with slopes 1/4 and -4
canadian
2012-02-23 21:42:10
slope of 1/4 and -4
slope of 1/4 and -4
PChang
2012-02-23 21:42:10
4
4
Yongyi781
2012-02-23 21:42:10
Slopes 1/4 and -4*
Slopes 1/4 and -4*
fractals
2012-02-23 21:42:10
1/4 and -4 as slopes
1/4 and -4 as slopes
copeland
2012-02-23 21:42:17
What's the value of f for this guy?
What's the value of f for this guy?
ABCDE
2012-02-23 21:42:49
1
1
canadian
2012-02-23 21:42:49
1
1
Yongyi781
2012-02-23 21:42:49
1
1
sammyMaX
2012-02-23 21:42:49
1
1
PChang
2012-02-23 21:42:49
1
1
fishinsea
2012-02-23 21:42:49
1
1
joshxiong
2012-02-23 21:42:49
1.
1.
nackster12
2012-02-23 21:42:49
1
1
Iggy Iguana
2012-02-23 21:42:49
1.
1.
duketip10
2012-02-23 21:42:49
1
1
fractals
2012-02-23 21:42:49
1
1
aopsvd
2012-02-23 21:42:49
1, isosceles
1, isosceles
bookfan
2012-02-23 21:42:49
1
1
copeland
2012-02-23 21:42:53
It's 1. This doesn't contribute to the product either.
It's 1. This doesn't contribute to the product either.
copeland
2012-02-23 21:42:56
So what's the answer?
So what's the answer?
aopsvd
2012-02-23 21:43:59
625/144
625/144
vcez
2012-02-23 21:43:59
B
B
Iggy Iguana
2012-02-23 21:43:59
(b)
(b)
tc1729
2012-02-23 21:43:59
B) 625/144
B) 625/144
KeepingItReal
2012-02-23 21:43:59
625/144
625/144
fractals
2012-02-23 21:43:59
1/(144/625) = 625/144
1/(144/625) = 625/144
fractals
2012-02-23 21:43:59
B
B
mdecross
2012-02-23 21:43:59
625/144, (B) (multiply the 6 into 24, take reciprocal)
625/144, (B) (multiply the 6 into 24, take reciprocal)
alex31415
2012-02-23 21:43:59
625/144?
625/144?
vcez
2012-02-23 21:43:59
$B$
$B$
numbertheorist17
2012-02-23 21:44:00
625/144
625/144
numbertheorist17
2012-02-23 21:44:00
(B) 625/144
(B) 625/144
copeland
2012-02-23 21:44:07
We want the reciprocal of this, so the actual answer is 625/144 and that is (B).
We want the reciprocal of this, so the actual answer is 625/144 and that is (B).
copeland
2012-02-23 21:44:10
Super!
Super!
copeland
2012-02-23 21:44:21
Phew.
Phew.
mcdonalds106_7
2012-02-23 21:45:09
what about slope 2/3?
what about slope 2/3?
copeland
2012-02-23 21:45:41
Nice catch.
Nice catch.
Iggy Iguana
2012-02-23 21:45:59
those don't contribute either
those don't contribute either
MathForFun
2012-02-23 21:45:59
1!
1!
PChang
2012-02-23 21:45:59
1
1
freddylukai
2012-02-23 21:45:59
cancels with the other one
cancels with the other one
mcdonalds106_7
2012-02-23 21:45:59
its isosceles though
its isosceles though
sammyMaX
2012-02-23 21:45:59
Tangent of that is one!
Tangent of that is one!
nackster12
2012-02-23 21:45:59
its still 1 though
its still 1 though
sammyMaX
2012-02-23 21:45:59
(we were lucky)
(we were lucky)
fractals
2012-02-23 21:45:59
It is 1, though.
It is 1, though.
mdecross
2012-02-23 21:45:59
...but still 1.
...but still 1.
copeland
2012-02-23 21:46:10
Good job! I totally missed that one.
Good job! I totally missed that one.
copeland
2012-02-23 21:46:11
That triangle has f(t)=1 as well, so it doesn't contribute either.
That triangle has f(t)=1 as well, so it doesn't contribute either.
copeland
2012-02-23 21:46:51
Alright I've kept you around for just about 3 hours. Usually we pick off a problem or two from earlier in the exams, but usually we're done around an hour ago.
Alright I've kept you around for just about 3 hours. Usually we pick off a problem or two from earlier in the exams, but usually we're done around an hour ago.
copeland
2012-02-23 21:47:17
So I think we'll shut this down and everyone can go to bed and sleep and be happy and dream of not math for a little while.
So I think we'll shut this down and everyone can go to bed and sleep and be happy and dream of not math for a little while.
copeland
2012-02-23 21:47:34
Oh, incidentally, our own Richard Rusczyk has been making video solutions for many of this year's AMC 10A and 12A problems. These will be up on our website in the next few days. Click on the "Videos" tab of the website to see them when they're up (and of course we'll make an announcement on the Community when they are available). They will probably be on YouTube too.
Oh, incidentally, our own Richard Rusczyk has been making video solutions for many of this year's AMC 10A and 12A problems. These will be up on our website in the next few days. Click on the "Videos" tab of the website to see them when they're up (and of course we'll make an announcement on the Community when they are available). They will probably be on YouTube too.
copeland
2012-02-23 21:47:42
If you qualify for the AIME, congratulations! AoPS will be holding our annual Special AIME Problem Seminar on the weekend of March 3 and 4, from 3:30 to 6:30 (Eastern Time) both days, taught by Richard Rusczyk and Dave Patrick. Each day consists of general approaches and important facts needed for problems within a given subject area, followed by a discussion of specific problems from past AIME competitions, or from other contests of a similar difficulty level. This course is largely a repeat of the weekend seminar we offered in 2010, and contains some material from the AIME Problem Series A course. This course is appropriate for students who are hoping to pass the AIME and qualify for the USA(J)MO. If a student already consistently scores above 10 on the AIME, this class is probably not necessary, and if a student is unlikely to answer more than 1 or 2 questions correctly, then that student should start with some of our Introduction series of classes. The cost is $75. You can enroll by clicking on the "School" tab of the website, and selecting "Special AIME Problem Seminar" from the list of courses.
If you qualify for the AIME, congratulations! AoPS will be holding our annual Special AIME Problem Seminar on the weekend of March 3 and 4, from 3:30 to 6:30 (Eastern Time) both days, taught by Richard Rusczyk and Dave Patrick. Each day consists of general approaches and important facts needed for problems within a given subject area, followed by a discussion of specific problems from past AIME competitions, or from other contests of a similar difficulty level. This course is largely a repeat of the weekend seminar we offered in 2010, and contains some material from the AIME Problem Series A course. This course is appropriate for students who are hoping to pass the AIME and qualify for the USA(J)MO. If a student already consistently scores above 10 on the AIME, this class is probably not necessary, and if a student is unlikely to answer more than 1 or 2 questions correctly, then that student should start with some of our Introduction series of classes. The cost is $75. You can enroll by clicking on the "School" tab of the website, and selecting "Special AIME Problem Seminar" from the list of courses.
copeland
2012-02-23 21:47:47
We will also have Math Jams for each of the two AIME contests in March -- check the website for specific dates and times.
We will also have Math Jams for each of the two AIME contests in March -- check the website for specific dates and times.
copeland
2012-02-23 21:47:53
That's all for tonight -- thanks for coming!
That's all for tonight -- thanks for coming!
c0mbusti0n1295
2012-02-23 21:48:32
good night Mr. Copeland!
good night Mr. Copeland!
ss5188
2012-02-23 21:48:32
How did you do on the AMC?
How did you do on the AMC?
c0mbusti0n1295
2012-02-23 21:48:32
thanks for your help Samson!
thanks for your help Samson!
ahaanomegas
2012-02-23 21:48:32
Thank you very much for this, Mr. Copeland, Duelist, Ms. Joelnia, and TowersFreak2006!
Thank you very much for this, Mr. Copeland, Duelist, Ms. Joelnia, and TowersFreak2006!
freddylukai
2012-02-23 21:48:32
Thank you!
Thank you!
blabla4198
2012-02-23 21:48:32
cya
cya
fractals
2012-02-23 21:48:32
:bow:
:bow:
ilovepink
2012-02-23 21:48:32
thank you!
thank you!
cerberus88
2012-02-23 21:48:32
Thank you!!!! :bow:
Thank you!!!! :bow:
lucylai
2012-02-23 21:48:32
:bow:
:bow:
RotomPlasma
2012-02-23 21:48:32
Thanks!
Thanks!
Iggy Iguana
2012-02-23 21:48:32
thank you too!!!
thank you too!!!
timoteomo3
2012-02-23 21:48:32
thanks!
thanks!
nackster12
2012-02-23 21:48:32
thanks!
thanks!
cong989
2012-02-23 21:48:32
thanks!
thanks!
Goos
2012-02-23 21:48:32
thanks!
thanks!
quinntai1997
2012-02-23 21:48:32
Awesome, Thanks!
Awesome, Thanks!
pattycakechichi
2012-02-23 21:48:32
Thank you!!!
Thank you!!!
copeland
2012-02-23 21:48:35
Thanks everyone, that was awesome!
Thanks everyone, that was awesome!
sammyMaX
2012-02-23 21:49:25
There wasn't an AMC A Day math jam was there?
There wasn't an AMC A Day math jam was there?
copeland
2012-02-23 21:49:28
Transcripts are here:
Transcripts are here:
Copyright © 2024 AoPS Incorporated. This page is copyrighted material. You can view and print this page for your own use, but you cannot share the contents of this file with others.